帕德逼近和函数综合问题


\section{帕德逼近}

\href{https://zhuanlan.zhihu.com/p/92873681}{帕德逼近(Pade’s Approximant)}


\begin{liti}
(2018年全国3卷高考理科,难度: \score{3}{5})
已知函数$f(x)=(2+x+ax^2)\ln(1+x)-2x$.

(1)若$a=0$,证明:当$-1<x<0$时, $f(x)<0$;当$x>0$时, $f(x)>0$;

(2)若$x=0$是 $f(x)$的极大值点,求$a$.
\end{liti}
\begin{jieda}

\end{jieda}


\begin{liti}
(2016年高考全国2卷理科,难度: \score{3}{5})
(I)讨论函数$f(x)=\frac{x-2}{x+2}e^x$的单调性,并证明当$x>0$时, $(x-2)e^x+x+2>0$;

(II)证明:当$a\in[0,1)$时,函数$g(x)=\frac{e^x-ax-a}{x^2}
\ (x>0)$有最小值.设$g(x)$的最小值为$h(a)$,求函数$h(a)$的值域.
\end{liti}
\begin{jieda}

\end{jieda}


\begin{liti}
(2016年高考全国2卷理科,难度: \score{3}{5})
已知函数$f(x)\ (x\in \mathbb{R})$满足$f(-x)=2-f(x)$,若函数$y=\frac{x+1}{x}$与$y=f(x)$图像的交点为$(x_1,y_1),(x_2,y_2),\cdots,(x_m,y_m)$,则$\sum_{i=1}^{m}(x_i+y_i)=$ \hfill (\qquad)
\begin{tasks}[label=(\Alph*)](2)
\task $0$
\task $m$
\task $2m$
\task $4m$
\end{tasks}
\end{liti}
\begin{jieda}
显然$f(x)$与$y=\frac{x+1}{x}$均关于点$(0,1)$对称,则有$x_i+x'_i=0,y_i+y'_i=2$,于是$\sum_ {i=1}^{m}(x_i+y_i)=\sum_ {i=1}^{m}x_i+\sum_ {i=1}^{m}y_i=0+m=m$.
\end{jieda}

2014年辽宁高考数学

2014年四川高考数学


\begin{liti}
(2015年高考理科全国1卷,难度: \score{3}{5})
已知函数$f(x)=x^3+ax+\frac{1}{4},g(x)=-\ln x$.

(I)当$a$为何值时, $x$轴为曲线$y=f(x)$的切线;

(II)用$\min\{m,n\}$表示$m,n$中的最小值,设函数$h(x)=\min\{f(x),g(x)\}\ (x>0)$,讨论$h(x)$零点的个数.
\end{liti}
\begin{jieda}

\end{jieda}

\begin{liti}
(2022年苏州大学数学分析考研,难度: \score{3}{5})设$f(x)$为$[0,1]$上的正值连续函数,且
\[\int_{0}^{1} f(x) d x=2021, \quad \int_{0}^{1} f^{2}(x) d x=2022^2.\]
\begin{enumerate}
\item[(1)] 证明:对任意的正整数$n$,存在$[0,1]$的分划$0=x_0<x_1 <\cdots<x_n=1$,使得
\[\int_{x_{k-1}}^{x_{k}} f(x) d x=\frac{1}{n} \int_{0}^{1} f(x) d x,\ (k=1,2,\cdots,n).\]
\item[(2)] 利用以上结论证明$\lim _{n \rightarrow \infty} \frac{1}{n} \sum_{k=1}^{n} f\left(x_{k}\right)$存在并求其值.
\end{enumerate}
\end{liti}
\begin{jieda}
\begin{enumerate}
\item[(1)] 令$g_k(x)=\int_{x_{k-1}}^{x} f(t) dt\ (k=1,2,\cdots,n)$,注意到$f(x)>0$,则$g_k(x)$在$[x_{k-1},1]$上单调递增,且$g_k(x_{k-1})=0,g_k(1)\geqslant \frac{1}{n} \int_{0}^{1} f(x) d x$.依次取$k=1,2,\cdots,n$,由连续函数的介值定理可知,存在$[0,1]$的分划$0=x_0<x_1 <\cdots<x_n=1$,使得
\[\int_{x_{k-1}}^{x_{k}} f(x) d x=\frac{1}{n} \int_{0}^{1} f(x) d x,\ (k=1,2,\cdots,n).\]

\item[(2)] 由积分中值定理可知,存在$\xi_k\in (x_{k-1},x_k)$,使得
$$\frac{1}{n} \int_{0}^{1} f(x) d x=\int_{x_{k-1}}^{x_{k}} f(x) d x
=f(\xi_k)(x_{k}-x_{k-1}),$$
而$f(x)$在$[0,1]$上有界,则当$n\to +\infty$时, $x_{k}-x_{k-1}\to 0$.故对任意$\varepsilon>0$,存在正整数$N$和$\delta>0$,当$n>N$时, $x_{k}-x_{k-1}<\delta$,当$t\in (x_{k-1},x_k)$时,有$|x_k-t|< \delta$,此时有$|f(x_k)-f(t)|<\varepsilon$.于是
\begin{align*}
&\int_0^1{f\left( t \right) dt}\cdot \frac{1}{n}\sum_{k=1}^n{f\left( x_k \right)}-\int_0^1{f^2\left( t \right) dt}\\
&=\sum_{k=1}^n{\left[ f\left( x_k \right) \cdot \frac{1}{n}\int_0^1{f\left( t \right) dt} \right]}-\int_0^1{f^2\left( t \right) dt}\\
&=\sum_{k=1}^n{\left[ f\left( x_k \right) \int_{x_{k-1}}^{x_k}{f\left( t \right) dt} \right]}-\sum_{k=1}^n{\int_{x_{k-1}}^{x_k}{f^2\left( t \right) dt}}\\
&=\sum_{k=1}^n{\int_{x_{k-1}}^{x_k}{f\left( t \right) \left[ f\left( x_k \right) -f\left( t \right) \right] dt}}.
\end{align*}

\begin{align*}
\left| \sum_{k=1}^n{\int_{x_{k-1}}^{x_k}{f\left( t \right) \left[ f\left( x_k \right) -f\left( t \right) \right] dt}} \right| &\leqslant \sum_{k=1}^n{\int_{x_{k-1}}^{x_k}{f\left( t \right) \left| f\left( x_k \right) -f\left( t \right) \right|dt}}\\
&<\varepsilon \sum_{k=1}^n{\int_{x_{k-1}}^{x_k}{f\left( t \right) dt}}=\varepsilon \int_0^1{f\left( t \right) dt},
\end{align*}
因此
$$
\lim_{n\rightarrow \infty}\frac{1}{n}\sum_{k=1}^n{f}\left( x_k \right) =\frac{\int_0^1{f^2\left( t \right) dt}}{\int_0^1{f\left( t \right) dt}}=\frac{2022^2}{2021}.
$$
\end{enumerate}
\end{jieda}


\begin{liti}
(难度: \score{3}{5})

\end{liti}
\begin{jieda}

\end{jieda}

 

 

\begin{liti}
(2013年湖北高考,长郡中学2021年高三月考三,难度: \score{3}{5})
已知$a$为常数,函数$f(x)=x(\ln x-2ax)$有两个极值点$x_1,x_2\ (x_1<x_2)$,
则\hfill (\qquad)
\begin{tasks}[label=(\Alph*)](2)
\task $0<a<\frac{1}{4}$
\task $x_1+x_2<2$
\task $f(x_1)<0$
\task $f(x_2)>-\frac{1}{2}$
\end{tasks}
\end{liti}
\begin{jieda}
因为$f'(x)=\ln x+1-4ax\ (x>0)$,令$f'(x)=0$,解得$4a=\frac{\ln x+1}{x}$.

令$g(x)=\frac{\ln x+1}{x}$,则$g'(x)=\frac{-\ln x}{x^2}$. $g(x)$在$(0,1)$上单调递增, 在$(1,+\infty)$上单调递减.作出$y=4a$和$g(x)=\frac{\ln x+1}{x}$的大致图像.

当$0<a<\frac{1}{4}$时, $f'(x)=0$有两个根$x_1,x_2$,且$x_1<1<x_2$,当$a\to 0^+$时,由图像可知$x_2\to +\infty$,而$0<x_1<1$,则$x_1+x_2\to +\infty$.

又因为$f(x)$在$(0,x_1)$上递减,在$(x_1,x_2)$上递增,在$(x_2,+\infty)$上递减.

而$0<x_1<1<x_2$,故$f(x_1)<f(1)=-2a<0,f(x_2)>f(1)=-2a>-\frac{1}{2}$.

故选: ACD.
\end{jieda}

 

\begin{liti}
(难度: \score{3}{5})
已知$a$为常数,函数$f(x)=e^x(x-ae^x)$有两个极值点$x_1,x_2\ (x_1<x_2)$,
则\hfill (\qquad)
\begin{tasks}[label=(\Alph*)](2)
\task $a$的取值范围是$\left( 0,\frac{1}{2} \right)$
\task $a$的取值范围是$\left( -\infty,\frac{1}{2} \right)$
\task $f(x_1)<0$
\task $f(x_2)>-\frac{1}{2}$
\end{tasks}
\end{liti}
\begin{jieda}
由题意得$f'(x)=e^x(x+1-2ae^x)$,因为$f(x)$有两个极值点$x_1,x_2\ (x_1<x_2)$,所以$f'(x)=0$有两个变号零点.又$e^x>0$,故只需$x+1-2ae^x=0$,即$2a=\frac{x+1}{e^x}$有两个不等实根.

令$g(x)=\frac{x+1}{e^x}$,则$g'(x)=-\frac{x}{e^x}$. $g(x)$在$(-\infty,0)$上单调递增, 在$(0,+\infty)$上单调递减.作出$y=2a$和$g(x)=\frac{x+1}{e^x}$的大致图像.

当$0<a<\frac{1}{2}$时, $f'(x)=0$有两个根$x_1,x_2$,且$x_1<0<x_2$,当$a\to 0^+$时,由图像可知$x_2\to +\infty$,而$-1<x_1<0$,则$x_1+x_2\to +\infty$,则应有$x_1+x_2>0$.

又因为$f(x)$在$(-1,x_1)$上递减,在$(x_1,x_2)$上递增,在$(x_2,+\infty)$上递减.

而$-1<x_1<0<x_2$,故$f(x_1)<f(-1)=\frac{1}{e}\left( -1-\frac{a}{e} \right)<0,f(x_2)>f(0)=-a>-\frac{1}{2}$.

故选: ACD.
\end{jieda}


\begin{liti}
(清华丘成桐数学英才班,难度: \score{3}{5})
设$a,b,c,d$是正实数, $w$是复数,满足$aw^3 + bw^2 +cw+d= 0$.
证明:
\[|w|\leq\max\left\{\frac{b}{a},\frac{c}{b},\frac{d}{c}\right\}.\]
\end{liti}
\begin{jieda}
(李朝顺)令$f(w)=aw^3 + bw^2 +cw+d,f(w)=0$的三个根为$w_i\ (i=1,2,3)$.
若$w_i$是实数,则$w_i<0$.否则,
$$0=f(w_i)=aw_i^3+bw_i^2+cw_i+d>0,$$
矛盾.

下面分两种情况讨论:

(1) $w_i$均为实数.

假设存在某个$w_i$使得$|w_i|=-w_i>\max\left\{\frac{b}{a},\frac{c}{b},\frac{d}{c}\right\}$,则
$$-w_i>\frac{b}{a}\Rightarrow aw_i+b<0,\quad -w_i>\frac{d}{c}\Rightarrow cw_i+d<0,$$
故$f(w_i)=(aw_i+b)w_i^2+cw_i+d<0$,与$f(w_i)=0$矛盾.

(2)存在某个$w_i\notin \mathbb{R}$,根据虚根成对原理, $\overline{w_i}$也为根.
不妨设$w_2=\overline{w_3}\notin \mathbb{R}$,则$w_1\in \mathbb{R}\Rightarrow w_1<0$,由韦达定理可知$w_1w_2w_3=-\frac{d}{a}$,则
$$
\left| w_2 \right|^2=\left| w_3 \right|^2=w_2w_3=\frac{-\frac{d}{a}}{w_1}.
$$

若$|w_2|>\max\left\{\frac{b} {a},\frac{c}{b},\frac{d}{c}\right\}$,则
\begin{align*}
\frac{-\frac{d}{a}}{w_1} &=\left| w_2 \right|^2>\frac{b}{a}\cdot \frac{c}{b}=\frac{c}{a}\Rightarrow cw_1+d>0,\\
\frac{-\frac{d}{a}}{w_1} &=\left| w_2 \right|^2>\frac{c}{b}\cdot \frac{d}{c}=\frac{d}{b}\Rightarrow aw_1+b>0.
\end{align*}
故$f(w_1)=(aw_1+b)w_1^2+cw_1+d>0$,与$f(w_1)=0$矛盾.证毕.
\end{jieda}


\begin{liti}
(苏州大学2021-2022秋季学期数学分析I期末考试,难度: \score{3}{5})
试判断$\sin(x^{2022})$与$\sin^{2022}(x)$在$(-\infty,+\infty)$上的一致连续性并说明理由.
\end{liti}
\begin{jieda}
先下结论: $\sin(x^{2022})$不一致连续, $\sin^{2022}(x)$一致连续.

证明:要证明$\sin(x^{2022})$不一致连续,我们使用反证法.

假设$\sin(x^{2022})$在$(-\infty,+\infty)$上一致连续,则$\varepsilon>0$, $\exists\delta>0$,对于$\forall x',x''\in \ \mathbb{R}$且$|x'-x''|<\delta$,有
$$|\sin(x'^{2022})-\sin(x''^{2022})|<\varepsilon.$$

不妨设$n\in \mathbb{N}$,令
$$x'=\sqrt[2022]{n\pi},\quad x''=\sqrt[2022]{n\pi+\frac{\pi}{2}},$$
则$\sin(x'^{2022})=0,\sin(x''^{2022})=\pm1.$,那么当$\varepsilon<1$时,不等式成立.

但又有
\begin{align*}
|x'-x''|&=|\sqrt[2022]{n\pi}-\sqrt[2022]{n\pi+\frac{\pi}{2}}|\\
&=\frac{|x'^{2022}-x''^{2022}|}{|x'^{2021}+x'^{2020}x''+\cdots+x'x''^{2020}+x''^{2021}|}
<\frac{\frac{\pi}{2}}{|2022x''^{2021}|},
\end{align*}
当$n$充分大时,有$x''^{2021}\rightarrow+\infty$, 就有$|x'-x''|<\delta$成立,由此引出矛盾,故$\sin(x^{2022})$不一致连续.

下面证明$\sin^{2022}(x)$在$(-\infty,+\infty)$上一致连续,我们利用导函数有界的性质来解决此问题.

这里我们先证明一个引理:

若函数$f(x)$在$[a,b]$上可导,且$|f'(x)|\leqslant M$,则$|f(b)-f(a)|\leqslant M(b-a)$.

证明:利用Lagrange中值定理, $\exists \xi\in(a,b)$,使得$f'(\xi)=\frac{f(b)-f(a)}{b-a}$,又$|f'(\xi)|\leqslant M$,可知$|f(b)-f(a)|\leqslant M(b-a)$.

下利用引理证明$\sin^{2022}(x)$的一致连续性.由于
$$f'(x)=2022\sin^{2021}(x)\cos(x).$$
$f'(x)$必然有界,不妨设$|f'(x)|\leqslant M,M>0$,则$\forall x',x''\in(-\infty,+\infty)$,有
$$|f(x')-f(x'')|\leqslant M|x'-x''|<\varepsilon.$$
故取$\delta=\frac{\varepsilon}{M}$,则$\forall \varepsilon>0$, $\exists \delta=\frac{\varepsilon}{M}$,使得$\forall x',x''\in(-\infty,+\infty)$且$|x'-x''|<\delta$,有$|f(x')-f(x'')|<\varepsilon$.
\end{jieda}


\begin{liti}
(2020年东南大学数学分析,难度: \score{3}{5})
计算
$$\int_0^1 \frac{\mathrm{d}x}{\sqrt{1-x^4}}\int_0^1\frac{x^2\mathrm{d}x}{\sqrt{1-x^4}}.$$
\end{liti}
\begin{jieda}
注意到
$$\int_0^1 \frac{\mathrm{d}x}{\sqrt{1-x^4}}=\frac{1}{4}\int_0^1 (x^{4})^{\frac{1}{4}-1} (1-x^4)^{\frac{1}{2}-1}\mathrm{d}(x^4)=\frac{1}{4}\mathrm{B}\left(\frac{1}{4},\frac{1}{2}\right).$$
$$\int_0^1 \frac{x^2\mathrm{d}x}{\sqrt{1-x^4}}=\frac{1}{4}\int_0^1 (x^{4})^{\frac{3}{4}-1} (1-x^4)^{\frac{1}{2}-1}\mathrm{d}(x^4)=\frac{1}{4}\mathrm{B}\left(\frac{3}{4},\frac{1}{2}\right).$$
于是
$$\int_0^1 \frac{\mathrm{d}x}{\sqrt{1-x^4}}\int_0^1\frac{x^2\mathrm{d}x}{\sqrt{1-x^4}}=\frac{1}{4}\frac{\Gamma\left(\frac{1}{4}\right)\Gamma\left(\frac{1}{2}\right)\Gamma\left(\frac{3}{4}\right)\Gamma\left(\frac{1}{2}\right)}{\Gamma\left(\frac{3}{4}\right)\Gamma\left(\frac{5}{4}\right)}=\frac{1}{16}\frac{\Gamma\left(\frac{1}{4}\right)\Gamma^2\left(\frac{1}{2}\right)}{\frac{1}{4}\Gamma\left(\frac{1}{4}\right)}=\frac{\pi}{4}.$$
\end{jieda}

\begin{liti}
(2020年东南大学数学分析,难度: \score{3}{5})
计算
$$\int_{0}^{+\infty} \mathrm{e}^{-p x} \frac{\sin b x-\sin a x}{x} \mathrm{d} x, \quad p>0, b>a>0.$$
\end{liti}
\begin{jieda}
当$y\in [a,b]$时, $\displaystyle\left|\int_0^A\cos xy\mathrm{d}x\right|\leqslant \frac{2}{a}$,即$\displaystyle\int_0^A \cos xy\mathrm{d}x$关于$y\in [a,b]$一致有界; $\mathrm{e}^{-px}$关于$x$单调,且当$x\to+\infty$时, $\mathrm{e}^{-px}$关于$y$一致趋于$0$.由Dirichlet判别法, $\displaystyle\int_0^{+\infty}\mathrm{e}^{-px}\cos xy\mathrm{d}x$关于$y\in [a,b]$一致收敛,由积分次序交换定理
\begin{align*}
\int_{0}^{+\infty} \mathrm{e}^{-p x} \frac{\sin b x-\sin a x}{x} \mathrm{d} x&=\int_0^{+\infty}\mathrm{e}^{-px}\mathrm{d}x\int_{a}^b \cos xy\mathrm{d}y=\int_a^b \mathrm{d}y\int_0^{+\infty}\mathrm{e}^{-px}\cos xy\mathrm{d}x\\ &=\int_a^b \frac{p}{p^2+y^2}\mathrm{d}y=\left[\arctan \frac{y}{p}\right]_{y=a}^{y=b}\\
&=\arctan \frac{b}{p}-\arctan \frac{a}{p}.
\end{align*}
\end{jieda}


\begin{liti}
(2020年东南大学数学分析,难度: \score{3}{5})

\end{liti}
\begin{jieda}

\end{jieda}

\begin{liti}
(难度: \score{3}{5})
\hfill (\qquad)
\begin{tasks}[label=(\Alph*)](2)
\task $a$的取值范围是$\left( 0,\frac{1}{2} \right)$
\task $a$的取值范围是$\left( -\infty,\frac{1}{2} \right)$
\task $f(x_1)<0$
\task $f(x_2)>-\frac{1}{2}$
\end{tasks}
\end{liti}
\begin{jieda}

\end{jieda}


\begin{liti}
(2021年山东省预赛,难度: \score{3}{5})
设函数$f(x)=x^2+ax+b$,对于任意的$a,b\in \mathbb{R}$,总存在$t\in [0,4]$,使得$|f(t)|\geqslant m$成立,则实数$m$的最大值是\underline{\hspace{2cm}}.
\end{liti}
\begin{jieda}
设$|f(t)|$的最大值为$M$,依题意有$m\leqslant M$恒成立,
$f(0)=b,f(4)=16+4a+b,f\left(-\frac{a}{2}\right)=-\frac{a^2}{4}+b,\Delta=a^2-4b$.

(1)当$\Delta\leqslant 0$时,若$-\frac{a}{2}\leqslant 2$,则$M\geqslant 4$;若$-\frac{a}{2}> 2$,则$M>4$.

(2)当$\Delta> 0$时,若$-\frac{a}{2}<0$,则$M>8$;若$-\frac{a}{2}>4$,则$M>8$.
若$0\leqslant-\frac{a}{2}\leqslant 2$,则$M\geqslant 2$;若$2<-\frac{a}{2}<4$,则$M>2$.

综上所述, $m\leqslant 2$.
\end{jieda}

 


\begin{liti}
设$f(x)=ax^2+bx+c$, $a,b,c$为实数,如果对于所有适合$-1\leqslant x \leqslant 1$的$x$值,都有$-1\leqslant f(x) \leqslant 1$成立,则对这些$x$的值有$-4\leqslant 2ax+b\leqslant 4$. (匈牙利, 1914)
\end{liti}
\begin{jieda}

$$
\begin{cases}
f\left( -1 \right) =a-b+c\\
f\left( 0 \right) =c\\
f\left( 1 \right) =a+b+c
\end{cases}$$
可知
$$
\begin{cases}
a=\frac{f\left( 1 \right) +f\left( -1 \right)}{2}-f\left( 0 \right)\\
b=\frac{f\left( 1 \right) -f\left( -1 \right)}{2}\\
c=f\left( 0 \right).
\end{cases}$$

$$
g\left( x \right) =2ax+b=\left[ f\left( 1 \right) +f\left( -1 \right) -2f\left( 0 \right) \right] x+\frac{f\left( 1 \right) -f\left( -1 \right)}{2}
$$
为一次函数,最值在端点处取得.


\begin{align*}
-4 &\leqslant g\left( 1 \right) =\frac{3}{2}f\left( 1 \right) +\frac{1}{2}f\left( -1 \right) -2f\left( 0 \right) \leqslant 4,
\\
-4 &\leqslant g\left( -1 \right) =-\frac{1}{2}f\left( 1 \right) -\frac{3}{2}f\left( -1 \right) +2f\left( 0 \right) \leqslant 4
\end{align*}
可知对$-1\leqslant x \leqslant 1$,都有$-1\leqslant f(x) \leqslant 1$成立,则对这些$x$的值有$-4\leqslant 2ax+b\leqslant 4$.
\end{jieda}

此题的背景是契比雪夫多项式的马尔科夫定理:如果具有实系数的$n$次多项式
$$f(x)=a_0+a_1x+a_2x^2+\cdots +a_nx^n$$
对所有的$-1\leqslant x \leqslant 1$满足不等式$-1\leqslant f(x) \leqslant 1$.
那么它的导函数满足不等式
$-n^2\leqslant f'(x)\leqslant n^2$.

虽然背景是高等的,但解法只用到一次函数$g(x)=2ax+b$的单调性、取值的技巧和不等式的放缩运算.

 


\begin{liti}
(2010年高联,难度: \score{3}{5})
已知函数$f(x)=ax^3+bx^2+cx+d\ (a\neq 0)$,当$0\leqslant x\leqslant 1$时, $|f'(x)|\leqslant 1$,试求$a$的最大值.
\end{liti}
\begin{jieda}
$f'(x)=3ax^2+2bx+c$,由
$$
\begin{cases}
f'\left( 0 \right) =c,\\
f'\left( \frac{1}{2} \right) =\frac{3}{4}a+b+c,\\
f'\left( 1 \right) =3a+2b+c\\
\end{cases}
$$

$$3a=2f'\left( 0 \right)+2f'\left( 1 \right)-4f'\left( \frac{1}{2} \right).$$
所以
\begin{align*}
3\left| a \right|&=\left| 2f'\left( 0 \right) +2f'\left( 1 \right) -4f'\left( \frac{1}{2} \right) \right|\\
&\leqslant 2\left| f'\left( 0 \right) \right|+2\left| f'\left( 1 \right) \right|+4\left| f'\left( \frac{1}{2} \right) \right|\leqslant 8,
\end{align*}
故$a\leqslant \frac{8}{3}$.

又易知当$f(x)=\frac{8}{3}x^3-4x^2+x+m$ ($m$为常数)满足题设条件,所以$a$最大值为$\frac{8}{3}$.

\textbf{解法二.} $f'(x)=3ax^2+2bx+c$.

设$g(x)=f'(x)+1$,则当$0\leqslant x\leqslant 1$时, $0\leqslant g(x)\leqslant 2$.

设$z=2x-1$,则$x=\frac{z+1}{2},-1\leqslant z\leqslant 1$.
$$
h\left( z \right) =g\left( \frac{z+1}{2} \right) =\frac{3a}{4}z^2+\frac{3a+2b}{2}z+\frac{3a}{4}+b+c+1.
$$
容易知道当$-1\leqslant z\leqslant 1$时, $0\leqslant h(z)\leqslant 2,0\leqslant h(-z)\leqslant 2$.

从而当$-1\leqslant z\leqslant 1$时,$0\leqslant \frac{h(z)+h(-z)}{2}\leqslant 2$,即
$$0\leqslant \frac{3a}{4}z^2+\frac{3a}{4}+b+c+1\leqslant 2,$$
从而$\frac{3a}{4}+b+c+1\geqslant 0,\frac{3a}{4}z^2\leqslant 2$,

由$0\leqslant z^2\leqslant 1$知$a\leqslant \frac{8}{3}$.

又易知当$f(x)=\frac{8}{3}x^3-4x^2+x+m$ ($m$为常数)满足题设条件,所以$a$最大值为$\frac{8}{3}$.
\end{jieda}

 

\begin{liti}
(2011年安徽预赛,难度: \score{3}{5})
设$f(x)=ax^3+bx+c$ ($a,b,c$是实数),当$0\leqslant x\leqslant 1$时, $0\leqslant f(x)\leqslant 1$.求$b$的最大可能值.
\end{liti}
\begin{jieda}

$$\begin{cases}
f\left( 0 \right) =c,\\
f\left( 1 \right) =a+b+c,\\
f\left( \sqrt{\frac{1}{3}} \right) =\frac{a}{3\sqrt{3}}+\frac{b}{\sqrt{3}}+c
\end{cases}$$
可知
$$
2b=3\sqrt{3}f\left( \sqrt{\frac{1}{3}} \right) -f\left( 1 \right) -\left( 3\sqrt{3}-1 \right) f\left( 0 \right) \leqslant 3\sqrt{3}
$$
$f\left( x \right) =\frac{3\sqrt{3}}{2}\left( x-x^3 \right)$满足题设, $b$的最大可能值为$\frac{3\sqrt{3}}{2}$.
\end{jieda}


\begin{liti}
(2021年山东省预赛第二套,难度: \score{3}{5})
设实系数多项式$f(x)=ax^4+bx^3+cx^2+dx+e$,当$x\in [-1,1]$时,总有$|f(x)|\leqslant 1$,则$|c|$的最大值为\underline{\hspace{2cm}}.
\end{liti}
\begin{jieda}

$$
g\left( x \right) =\frac{f\left( x \right) +f\left( -x \right)}{2}=ax^4+cx^2+e,
$$
则对任意$x\in [0,1]$,都有$|g(x)|\leqslant 1$.于是
\begin{align*}
\left| c \right| &=\left| 4\left( \frac{a}{4}+\frac{c}{2}+e \right) -\left( a+c+e \right) -3e \right|\\
&=\left| 4g\left( \frac{\sqrt{2}}{2} \right) -g\left( 1 \right) -3g\left( 0 \right) \right|\leqslant 8.
\end{align*}
当$f(x)=8x^4-8x^2+1$时,令$x=\cos\theta$,则$f(x)=\cos4\theta$,于是$|f(x)|\leqslant 1$.

综上所述, $|c|_{\max}=8$.
\end{jieda}


\begin{liti}
(2020年东南大学数学分析,难度: \score{3}{5})

\end{liti}
\begin{jieda}

\end{jieda}



\begin{liti}
(难度: \score{3}{5})解方程
$$
\begin{cases}
x_1+x_2+\cdots +x_n=0,\\
x_{1}^{2}+x_{2}^{2}+\cdots +x_{n}^{2} =0,\\
\cdots \cdots \cdots\\
x_{1}^{n}+x_{2}^{n}+\cdots +x_{n}^{n} =0.
\end{cases}
$$
\end{liti}
\begin{jieda}

\end{jieda}

 

\begin{liti}
(难度: \score{3}{5})解方程
$$
\begin{cases}
x_1+x_2+\cdots +x_n=n,\\
x_{1}^{2}+x_{2}^{2}+\cdots +x_{n}^{2} =n,\\
\cdots \cdots \cdots\\
x_{1}^{n}+x_{2}^{n}+\cdots +x_{n}^{n} =n.
\end{cases}
$$
\end{liti}
\begin{jieda}
作以$x_1,x_2,\cdots,x_n$为根的多项式
\begin{align*}
f\left( x \right) &=\left( x-x_1 \right) \left( x-x_2 \right) \cdots \left( x-x_n \right)\\
&=x^n+a_{n-1}x^{n-1}+\cdots +a_1x+a_0.
\end{align*}

$$f\left( x_k \right)=x_k^n+a_{n-1}x_k^{n-1}+\cdots +a_1x_k+a_0=0,\quad
k=1,2,\cdots,n.$$
于是
\begin{align*}
\sum_{k=1}^n{f\left( x_k \right)} &=\sum_{k=1}^n{\left( x_{k}^{n}+a_{n-1}x_{k}^{n-1}+\cdots +a_1x_k+a_0 \right)}\\
&=\sum_{k=1}^n{x_{k}^{n}}+a_{n-1}\sum_{k=1}^n{x_{k}^{n-1}}+\cdots +a_1\sum_{k=1}^n{x_k}+na_0=0.
\end{align*}
由方程组知
$$n+a_{n-1}n+\cdots +a_1n+na_0=0,$$
从而
$$f(1)=1+a_{n-1}+\cdots +a_1+a_0=0.$$
这说明$x=1$为$f(x)$的一个根.由对称性,不妨设$x_n=1$,由原方程组得
$$
\begin{cases}
x_1+x_2+\cdots +x_{n-1}=n-1,\\
x_{1}^{2}+x_{2}^{2}+\cdots +x_{n-1}^{2} =n-1,\\
\cdots \cdots \cdots\\
x_{1}^{n}+x_{2}^{n}+\cdots +x_{n-1}^{n} =n-1.
\end{cases}
$$
同理可得$x_1,\cdots,x_{n-1}$中有一个为$1$.继续下去,必有
$$x_1=x_2=\cdots=x_n=1.$$
\end{jieda}

\begin{liti}
(2020年东南大学数学分析,难度: \score{3}{5})

\end{liti}
\begin{jieda}

\end{jieda}

 

 


\section{2019北京大学数学科学学院直博生摸底考试试题}

\begin{center}
(2019年4月6日9:00-11:00)
\end{center}

\begin{enumerate}
\item 设$a>0,a_1=a,a_2=a^a,a_3=a^{a^a},\cdots,a_{n+1}=a^{a_n},\cdots$,集合$S=\{a|\text{序列$\{a_n\}$极限存在}\}$.
\begin{enumerate}
\item 证明$S$有上界.
\item 求$S$的上确界.
\end{enumerate}

\textbf{分析:}显然当$a=2$时, $a_n$不收敛,则$S$有上界$a=2$;

以下求$\sup S$,显然当$a=1$时有$a_n\equiv 1$,这显然是收敛的,因此只需考虑$a>1$的情况,由蛛网理论可知, $a_n$一定收敛到曲线
$$f_1(x)=x,\qquad f_2(x)=a^x$$
的交点上,而当该交点为切点时, $a_n$收敛到$\sup S$,此时设切点横坐标为$x_0$,则
\begin{align*}
\begin{cases}
f_1(x_0)=f_2(x_0)\Rightarrow a^{x_0}=x_0, \quad (1) \\
f'_1(x_0)=f'_2(x_0)\Rightarrow a^{x_0}\ln a=1.\quad (2)
\end{cases}
\end{align*}
解出$a=e^{\frac{1}{x_0}}$,代入(1)式可得$a=e^{\frac{1}{e}}$,这就是$\sup S$;

事实上$\inf S=\frac{1}{e^e}$,此时有方程组
\begin{align*}
\begin{cases}
f_1(x_0)=f_2(x_0)\Rightarrow a^{x_0}=x_0,\quad (3) \\
f'_1(x_0)=-f'_2(x_0)\Rightarrow a^{x_0}\ln a=-1.\quad (4)
\end{cases}
\end{align*}
这就求得$S=\left[ \frac{1}{e^e},e^{\frac{1}{e}} \right]$.
蛛网理论的临界条件可参考各经济学著作.

 

\begin{jieda}
\begin{enumerate}
\item 设$g(x)=a^x-x$,则$g'(x)=a^x\ln a-1$.于是$g(x)$在$\left( -\infty ,\log _a\frac{1}{\ln a} \right)$上递减,在$\left(\log _a\frac{1}{\ln a},+\infty\right)$上递增,则$g(x)$在$x=\log _a\frac{1}{\ln a}$处取得最小值,且
$$
g\left( \log _a\frac{1}{\ln a} \right) =\frac{1}{\ln a}-\log _a\frac{1}{\ln a}=\frac{1}{\ln a}-\frac{\ln \frac{1}{\ln a}}{\ln a}=\frac{1+\ln\ln a}{\ln a}.
$$
当$a>e^{\frac{1}{e}}$时, $\ln\ln a>-1$,此时$g\left( \log _a\frac{1}{\ln a} \right)>0$,此时$g(x)>0$对任意$x\in \mathbb{R}$成立.
$g$没有零点, $f$无不动点.


$$a_{n+1}-a_n=f(a_n)-a_n=g(a_n)>0,$$
故$a_n$递增.若$\{a_n\}$有界,则由单调有界定理可知$\lim_{n\to\infty}a_n=a$存在,此时$f(a)=a$,即$f$具有不动点,矛盾.故$\{a_n\}$无界,是正无穷大量.故$S$有上界$e^{\frac{1}{e}}$.


\item 当$a=e^{\frac{1}{e}}$时, $g(x)$仅有一个零点$\log _a\frac{1}{\ln a}=\frac{\ln \frac{1}{\ln a}}{\ln a}=e$, $f$仅有唯一的不动点$e$.注意到
$$a_{n+1}-a_n=f(a_n)-a_n=g(a_n)=\begin{cases}
\geqslant 0, & a_n\leqslant e, \\
\leqslant 0, & a_n\geqslant e.
\end{cases},$$
我们知当$a_1=e^{\frac{1}{e}}<e$时, $a_2-a_1=g(a_1)\geqslant 0,a_2-e=f(a_1)-f(e)\leqslant 0$;

$a_3-a_2=g(a_2)\geqslant 0,a_3-e=f(a_2)-f(e)\leqslant 0$;如此类推,可知$\{a_n\}$递增有上界$e$,则极限存在,且收敛于$f$的唯一的不动点$e$.这就证明了$S$的上确界为$e^{\frac{1}{e}}$.
\end{enumerate}
\end{jieda}

 

\item \begin{enumerate}
\item 设$y=\arctan \frac{3 x\left(x^{2}-1\right)}{x^{4}-4 x^{2}+1}$,求$\frac{dy}{dx}$.

\item 求定积分$\int_{0}^{1} \frac{1+x^{4}}{1+x^{6}} d x$.
\end{enumerate}
\begin{jieda}

\end{jieda}

\item 仿照定积分的第二中值定理,严格叙述第一型曲线积分的第二中值定理,并证明.
\begin{jieda}

\end{jieda}


\item $f(x),g(x)$在$\mathbb{R}$上连续, $g(x)$以$T>0$为周期.证明: $f(f(x))=-x^3+g(x)$无连续解.
\begin{jieda}

\end{jieda}


\item 在单位圆盘$D$内, $u=\Delta u,u|_{\partial D}=0$.证明: $u\equiv 0$.
%Jordan边界就好.
\begin{jieda}

\end{jieda}


\item 设$a_0,a_1,a_2$为有理数,且$\left| \begin{array}{ccc}{a_{0}} & {a_{1}} & {a_{2}} \\ {a_{2}} & {a_{0}+a_{1}} & {a_{1}+a_{2}} \\ {a_{1}} & {a_{0}+a_{2}} & {a_{0}+a_{1}}\end{array}\right|=0$.证明: $a_0=a_1=a_2=0$.

订正: $\left| \begin{array}{ccc}{a_{0}} & {a_{1}} & {a_{2}} \\ {a_{2}} & {a_{0}+a_{1}} & {a_{1}+a_{2}} \\ {a_{1}} & a_{2} & {a_{0}+a_{1}}\end{array}\right|=0$

\[
\det \left( \begin{matrix}
a_0& a_1& a_2\\
a_2& a_0+a_1& a_1+a_2\\
a_1& a_2& a_0+a_1\\
\end{matrix} \right) =\det \left( \begin{matrix}
a_0& a_1& a_2\\
a_2-a_0& a_0& a_1\\
a_1& a_2& a_0+a_1\\
\end{matrix} \right).\]

\item 设实二次型$f\left(x_{1}, x_{2}, x_{3}\right)=4 x_{1} x_{2}-2 x_{1} x_{3}+3 x_{2}^{2}+2 x_{2} x_{3}$.
\begin{enumerate}
\item 将其写成$f(x)=x^TAx$ ($A$为对称矩阵)的形式,并求出其特征值,特征向量;

\item 求正交矩阵$P$和对角矩阵$D$,使得$A=PDP^T$;

\item 若三维向量$x$满足$\|x\|=1$,求$f(x)$的最大值.
\end{enumerate}
\begin{jieda}
\begin{enumerate}
\item $f$对应的矩阵为
$$
A=\left( \begin{matrix}
0& 2& -1\\
2& 3& -2\\
-1& -2& 0\\
\end{matrix} \right),
$$
其特征值为$5,-1,-1$.


$$
5E-A\rightarrow \left( \begin{matrix}
1& 0& 1\\
0& 1& 2\\
0& 0& 0
\end{matrix} \right)
$$
知$A$的属于特征值$5$的特征向量为
$$
\xi _1=\left( -1,-2,1 \right) ^{\text{T}}.
$$


$$
-E-A\rightarrow \left( \begin{matrix}
1& 2& -1\\
0& 0& 0\\
0& 0& 0
\end{matrix} \right)
$$
知$A$的属于特征值$-1$的特征向量为
$$
\xi _2=\left( -2,1,0 \right) ^{\text{T}},\quad \xi _3=\left( 1,0,1 \right) ^{\text{T}}.
$$

\item 将$\xi _1,\xi _2,\xi _3$标准正交化为$\eta_1,\eta_2,\eta_3$,并设
$$
P=\left( \eta _1,\eta _2,\eta _3 \right) =\left( \begin{matrix}
-\frac{1}{\sqrt{6}}& -\frac{2}{\sqrt{5}}& \frac{1}{\sqrt{30}}\\
-\frac{2}{\sqrt{6}}& \frac{1}{\sqrt{5}}& \frac{2}{\sqrt{30}}\\
\frac{1}{\sqrt{6}}& 0& \frac{5}{\sqrt{30}}\\
\end{matrix} \right),
$$
则$A=PDP^T$,其中$D=\text{diag}\left\{ 5,-1,-1 \right\}$.

\item
\begin{align*}
f\left( x \right) &=x^{\text{T}}Ax=y^{\text{T}}Dy=5y_{1}^{2}-y_{2}^{2}-y_{3}^{2}\\
&=5y_{1}^{2}-\left( 1-y_{1}^{2} \right) =6y_{1}^{2}-1\leqslant 5,
\end{align*}
且等号成立当且仅当$y_1=1,y_2=y_3=0\Leftrightarrow y=e_1\Leftrightarrow$ $x$是$A$的属于特征值$5$的单位特征向量$\eta_1$ (也就是$P$的第一列).

故$f(x)$的最大值为$5$.
\end{enumerate}
\end{jieda}


\item 设矩阵$A$的列数$=B$的行数$=n$,证明$r(A)+r(B)=r(AB)+n$当且仅当$A$的解空间$\subseteq B$的列空间.
\begin{jieda}
设数域为$\mathbb{P}$.定义
\begin{align*}
\mathscr{A}\left( e_1,\cdots ,e_n \right) &=\left( e_1,\cdots ,e_m \right) A,\\
\mathscr{B}\left( e_1,\cdots ,e_p \right) &=\left( e_1,\cdots ,e_n \right) B,
\end{align*}

$$\mathscr{A}:\mathbb{P}^n\to \mathbb{P}^m,\quad \mathscr{B}:\mathbb{P}^p\to \mathbb{P}^n.$$
考虑
$$\mathscr{A}|_{\mathrm{Im} \mathscr{B}}:\mathrm{Im} \mathscr{B}\to \mathbb{P}^m,$$
我们有
$$\dim \mathrm{Im} \mathscr{B}=\dim\mathscr{A}|_{\mathrm{Im} \mathscr{B}}(\mathrm{Im} \mathscr{B})+\dim\mathrm{Ker}(\mathscr{A}|_{\mathrm{Im} \mathscr{B}}).$$
由$\dim \mathscr{B}=\mathrm{rank}(B)$, $\dim\mathscr{A}|_{\mathrm{Im} \mathscr{B}}(\mathrm{Im} \mathscr{B})=\dim\mathscr{A}\mathscr{B}(\mathbb{P}^p)=\mathrm{rank}(AB)$,
$\dim\mathrm{Ker}(\mathscr{A}|_{\mathrm{Im} \mathscr{B}})=
\text{dim}\left\{ \alpha \in \mathbb{P}^n:\alpha \in \text{Im}\mathscr{B},\alpha \in \text{Ker}\mathscr{A} \right\} =\text{dim}\left( \text{Im}\mathscr{B}\cap \text{Ker}\mathscr{A} \right)$
可知
\begin{align*}
\mathrm{rank}(B)&=\mathrm{rank}(AB)+\text{dim}\left( \text{Im}\mathscr{B}\cap \text{Ker}\mathscr{A} \right)\\
&\leqslant \mathrm{rank}(AB)+\dim\text{Ker}\mathscr{A}\\
&=\mathrm{rank}(AB)+n-\mathrm{rank}(A),
\end{align*}
且等号成立当且仅当$\mathrm{rank}(B)=\mathrm{rank}(AB)+n-\mathrm{rank}(A)$,
当且仅当$\text{Ker}\mathscr{A}\subset \text{Im}\mathscr{B}$, 当且仅当$A$的解空间$\{x:Ax=0\}$为$B$的列空间的子空间.
\end{jieda}


\item $A$为$M_n(\mathbb{C})$中的幂零矩阵, $M_n(\mathbb{C})$上的线性变换$T_A:B\mapsto AB-BA$.证明:若$(T_A)^2B=0$,则$AB$是幂零矩阵.
\begin{jieda}
(1)先证一个结论.若$A$为幂零矩阵,则对任意的正整数$k$,有$\mathrm{tr}(A^k)=0$;反之,若对任意的$1\leqslant k\leqslant n$,均有$\mathrm{tr}(A^k)=0$,则$A$为幂零矩阵.

事实上, $A$是幂零矩阵, 则存在$m\geqslant 1$,使得$A^m=0$, 则$A$的特征值全为$0$, 则对任意$k\geqslant 1$,有$A^k$的特征值全为$0$,则$\mathrm{tr}(A^k)=0$.


设$A$的特征多项式为
$$f(\lambda)=|\lambda E-A|=\lambda^n-\sigma_1\lambda^{n-1}+\cdots+(-1)^n\sigma_n
=(\lambda-\lambda_1)\cdots (\lambda-\lambda_n).$$
则$A$的特征值为$\lambda_1,\cdots,\lambda_n$.而对任意$k\geqslant 1$, $A^k$的特征值为$\lambda_1^k,\cdots,\lambda_n^k$.于是
$$
\begin{cases}
\lambda _1+\cdots +\lambda _n=\text{tr}A=0,\\
\lambda _{1}^{2}+\cdots +\lambda _{n}^{2}=\text{tr}\left( A^2 \right) =0,\\
\cdots \cdots \cdots\\
\lambda _{1}^{n}+\cdots +\lambda _{n}^{n}=\text{tr}\left( A^n \right) =0,
\end{cases}
$$
由牛顿定理可知$\sigma_1=\sigma_2=\cdots=\sigma_n=0$.故$f(\lambda)=\lambda^n$.
由Hamilton-Cayley定理可知$A^n=0$.这就证明了$A$是幂零的.

(2)设$C=T_AB=AB-BA$,则由$(T_A)^2B=0$可知$AC=CA$.于是对任意$k\geqslant 1$,有
\begin{align*}
C^k &=C^{k-1}(AB-BA)=C^{k-1}AB-C^{k-1}BA\\
&=A(C^{k-1}B)-(C^{k-1}B)A,
\end{align*}
故$\mathrm{tr}(C^k)=0$, $C$是幂零的,故$C$的特征值全为$0$.

(3)由$C=AB-BA$得$AB=BA+C$.再由$A^kB=BA^k+kCA^{k-1}$可知
$A^{k+1}B=ABA^k+kACA^{k-1}=(BA+C)A^k+kACA^{k-1}=BA^{k+1}+(k+1)CA^k$.

由数学归纳法可知,对任意$k\geqslant 0$,有
\begin{align*}
A^{k+1}B=BA^{k+1}+(k+1)CA^k.\tag{1}
\end{align*}

(4)为证$AB$幂零,只需证明$AB$的任一特征值$\lambda=0$.

利用反证法,假设$\alpha\neq 0$是$AB$的属于特征值$\lambda\neq 0$的特征向量,则
\begin{align*}
AB\alpha=\lambda\alpha,\quad A^k(\lambda\alpha)=BA^{k+1}\alpha+(k+1)CA^k\alpha.\tag{2}
\end{align*}
由$A$幂零知存在$m\geqslant 1$使得
$$A\neq 0,\cdots,A^{m-1}\neq 0,A^m=0.$$
在(2)中取$k=m-1$得$\lambda A^{m-1}\alpha=mCA^{m-1}\alpha$,则
$C(CA^{m-1}\alpha)=\frac{\lambda}{m}A^{m-1}\alpha$.

由$A^{m-1}\alpha\neq 0$可知$\frac{\lambda}{m}\neq 0$是$C$的特征值,与$C$幂零矛盾!
故$A^{m-1}\alpha= 0$.在(2)中取$k=m-2$得
$$\lambda A^{m-2}\alpha=(m-1)CA^{m-2}\alpha.$$
如此类推, $A^{m-2}\alpha= 0,\cdots,\alpha= 0$.这与$\alpha\neq 0$是$AB$的特征向量矛盾.故有结论$\lambda=0$.
\end{jieda}

\item 在$y=0$平面上的圆$(x-2)^2+z^2=1$绕$z$轴旋转成一环面.
\begin{enumerate}
\item 求这个环面的一个参数方程;

\item 它的正等测投影(即在平面$x+y+z=0$上的垂直投影)外周是否为椭圆?
\end{enumerate}
\begin{jieda}
\begin{enumerate}
\item 环面$S$上的点$(X,Y,Z)$由$y=0$上的圆$(x-2)^2+z^2=1$中的点$(x,0,z)$绕$z$轴旋转得到,而
$$X^2+Y^2=x^2,\quad Z=z.$$
令$X=x\cos\theta,Y=x\sin \theta$,则
$$Z=z=\pm \sqrt{4x-x^2-3}.$$
由此可得环面的一个参数方程表示
$$X=x\cos\theta,\quad Y=x\sin \theta,\quad Z=\pm \sqrt{4x-x^2-3},\quad -1\leqslant x\leqslant 3.$$


\item 由(1)可知环面$S$的方程为
$$1=(x-2)^2+z^2=x^2-4x+4+z^2,(x^2+z^2+3)^2=16x^2,$$
$$\left(X^2+Y^2+Z^2+3\right)^2=16\left(X^2+Y^2\right).$$
将$\xi_1=\frac{1}{\sqrt{3}}(1,1,1)^{\mathrm{T}}$扩充为$\mathbb{R}^3$的一组标准正交基(就是对$x_1+x_2+x_3=0$的基础解系标准正交化)
$$\xi_1=\frac{1}{\sqrt{3}}(1,1,1)^{\mathrm{T}},\quad \xi_2=\frac{1}{\sqrt{2}}(-1,1,0)^{\mathrm{T}},\quad
\xi_3=\frac{1}{\sqrt{6}}(-1,-1,2)^{\mathrm{T}}.$$
令$P=(\xi_1,\xi_2,\xi_3)$,
$$
\left( \begin{array}{c}
X\\
Y\\
Z\\
\end{array} \right) =P^{\text{T}}\left( \begin{array}{c}
u\\
v\\
w\\
\end{array} \right),
$$
则$S$化为
$$\left(u^2+v^2+w^2+3\right)^2=\frac{8}{3}\left(5u^2-2uv+5v^2+4uw+4vw+2w^2\right).$$
令$u=0$,得$S$的正等测投影外周的方程为
$$\left(v^2+w^2+3\right)^2=\frac{8}{3}\left(5v^2+4vw+2w^2\right).$$
设$A=\left( \begin{matrix}
5& 2\\
2& 2\\
\end{matrix} \right)$,则$A$的特征值为$6,1$.故经过适当的正交变换后,上式变为
$$\left(V^2+W^2+3\right)^2=\frac{8}{3}\left(6V^2+W^2\right).$$
这不是一个椭圆.
\end{enumerate}
\end{jieda}
\end{enumerate}

 


 

 

 

\section{复旦大学2007年数学竞赛分析卷试题}

1.求
$$
\lim_{n\rightarrow +\infty}\left( \tan \frac{1}{n^2}+\tan \frac{3}{n^2}+\cdots +\tan \frac{2n-1}{n^2} \right).
$$


2.设数列$x_n\geqslant 0$满足
$$x_{m+n+k}\leqslant x_m\cdot x_n\cdot x_k,\quad \forall m,n,k\geqslant 1.$$
证明: $\sqrt[n]{x_n}$的极限存在.

3.设$f(x),\varphi(x)$在闭区间$[a,b]$上连续,且$f>0,\varphi>0$.证明:
$$
\lim_{n\rightarrow +\infty}\frac{\int_a^b{\varphi \left( x \right) f^{n+1}\left( x \right) dx}}{\int_a^b{\varphi \left( x \right) f^n\left( x \right) dx}}=\max_{x\in \left[ a,b \right]}\left| f\left( x \right) \right|.
$$

4.设$f(\cdot)$在$[a,b]$上连续,但不为常数.求证: $\exists\xi\in(a,b)$使$f(\cdot)$在点$\xi$不取极值.


5.设$\sum_{n=1}^{\infty}u_n(x)$关于$x\in(a,b)$一致收敛, $\lim_{x\to a^+}u_n(x)$存在,则$\sum_{n=1}^{\infty}\lim_{x\to a^+}u_n(x)$收敛,且
$$\lim_{x\to a^+}\sum_{n=1}^{\infty}u_n(x)=\sum_{n=1}^{\infty}\lim_{x\to a^+}u_n(x).$$

6.设$f(x)$在$[0,2\pi]$上是Lebesgue可积的.证明:
$$\lim_{n\rightarrow \infty}\int_{0}^{2\pi}f(x)e^{inx}dx=0.$$

7.设$\Omega$是复平面上的连通开集,给定$\omega\in\Omega$,定义$\Omega$上的Dirichlet空间,
$$
\mathcal{L}=\left\{ f\in \text{Hol}\left( \Omega \right) :\lVert f \rVert ^2=\left| f\left( \omega \right) \right|^2+\int_{\Omega}{\left| f'\left( z \right) \right|^2dA\left( z \right)}<\infty \right\},
$$
这里, $\text{Hol}\left( \Omega \right)$表示$\Omega$上解析函数全体, $dA(z)$是面积测度.证明:

(1) $\mathcal{L}$是一个Hilbert空间,并写出内积;

(2)对每个$\lambda\in \Omega$,赋值泛函$f\mapsto f(\lambda)$是连续的.由Riesz表示定理,存在唯一的$K_{\lambda}\in \mathcal{L}$使得$f(\lambda)=\langle f,K_{\lambda}\rangle$,当$\Omega$是开单位圆盘$\{z:|z|<1\}$时,写出$K_{\lambda}$.


8.设$\mathcal{A}$是由一些集合所成的集类.

(1)证明$\mathcal{A}$是(实分析意义下的)环当且仅当它对二个集合的对称差运算和交运算封闭.此时$\mathcal{A}$按对称差运算和交运算成为(抽象代数意义下的)交换环:将对称差视为加法而交视为乘法.


(2)集合$\mathbb{Z}_2$只有两个元素$[0]$和$[1]$,它按$[x]+[y]=[x+y]$及$[x][y]=[xy]$成为有单位元的交换环.从集合$X$至$\mathbb{Z}_2$的函数全体$\mathbb{Z}_2^X$自然是个交换的环.试作个环同构$f:2^X\to \mathbb{Z}_2^X$.


%http://www.doc88.com/p-3167137139848.html

 

\section{复旦大学2007年数学竞赛代数卷试题}

学校:\quad 院系:

姓名:\quad 学号:\quad 专业: \quad

1.设$A$是$m\times n$矩阵, $B$是$m\times s$矩阵, $C=(A|B)$是$A,B$拼成的矩阵,证明矩阵方程$AX=B$有解的充分必要条件是$R(A)=R(C)$, $R$表示矩阵的秩.


2.设$V=\mathbb{M}_n(\mathbb{R})$是$n$阶实方阵空间, $W\subset V$是由所有形如$CD-DC\ (C,D\in \mathbb{M}_n(\mathbb{R}))$的矩阵张成的子空间,试求$\dim W$ (要求写出计算依据).


3.若$A$是$n$阶方阵, $A^\ast$是其伴随矩阵$(n\geqslant 2)$,证明
$$R(A^\ast)=\begin{cases}
n, & R(A)=n; \\
1, & R(A)=n-1; \\
0, & R(A)<n-1.
\end{cases}$$

4.设实矩阵$X=\left( \begin{matrix}
A& B\\
C& D\\
\end{matrix} \right)$
其中$A,B,C,D$是n阶方阵且它们两两可以交换(关于矩阵乘法),证明$|X|\neq 0$当且仅当$|AD-BC|\neq 0$.

5.设$V_1,V_2$是有限维线性空间$V$的两个子空间,且
$$\dim(V_1+V_2)=\dim(V_1\cap V_2)+1$$
证明必有$V_1\subset V_2$或$V_2\subset V_1$.

6.设$A$是复数域$\mathbb{C}$上的$n$阶方阵,证明$A$可对角化当且仅当$R(A-aI_n)=R((A-aI_n)^2),\forall a\in \mathbb{C}$, $I_n$表示$n$阶单位阵.

 

7.设$A=(a_{i,j})_{n\times n},B=(b_{i,j})_{n\times n}$都是正定阵,证明$C=(a_{i,j}b_{i,j})_{n\times n}$也正定.

8.设$\mathbb{K}$是数域,证明$\mathbb{K}^n$的任一真子空间均是若干个$n-1$维子空间的交.

9.设$A,B,C$分别是数域$\mathbb{K}$上的$m\times n,n\times p,p\times q$矩阵,证明:
$$R(AB)+R(BC)\leqslant R(B)+R(ABC).$$

 

\section{复旦大学2008年数学竞赛分析卷试题}


1.数列$\{a_n\}$满足$0<a_n\leqslant a_{2n}+a_{2n+1}$,证明: $\sum_{n=1}^{\infty}a_n$发散.

 

2.设$-1<a_0<1,a_n=\sqrt{\frac{1+a_{n-1}}{2}}\ (n>0)$,问极限$\lim_{n\to\infty}4^n(1-a_n)$是否存在?存在的话,求出极限值.


3.计算Fibonacci数列的通项$a_n:a_1=1,a_2=1,a_{n+1}=a_n+a_{n-1}\ (n\geqslant 2)$.


4.设$P_n(x)=\sum_{k=0}^{n}\frac{x^k}{k!}$,证明: $P_n(x)P_{n+1}(x)$有唯一实根.


5. (1)求$I(\alpha)=\int_{0}^{1} |\alpha x-1|dx$关于$\alpha\in[0,+\infty)$的最小值.

(2)设$\int_{0}^{1}f(x)dx=1,\int_{0}^{1}xf(x)dx=0$,证明: $\sup_{x\in [0,1]}|f(x)|\geqslant \sqrt{2}+1$.


6.计算二重积分:
$$
\iint_D{\frac{\left( x+y \right) \ln \left( 1+\frac{y}{x} \right)}{\sqrt{1-x-y}}dxdy},
$$
其中区域$D$由直线$x+y=1$与两坐标轴所围三角形区域.


7. $f(x)$在$[0,1]$上连续,求极限:
$\lim_ {n\to\infty}n\int_{0}^{1}\frac{f(x)}{1+n^2x^2}dx$ (需要证明).


8.求$x\to 1_-$时,与$\sum_{n=0}^{\infty}x^{n^2}$等价的无穷大量.


%https://www.doc88.com/p-674130308510.html?r=1

\section{复旦大学2008年数学竞赛代数卷试题}

一、以$M_n(K)$表示数域$K$上的$n$阶方阵全体.设$\phi:M_n(K)\to K$是一个线性映射,满足

(1) $\phi(AB)=\phi(BA)$\ ($\forall A$、$B\in M_n(K)$)和

(2) $\phi(I_n)=n$,其中$I_n$是$n$阶单位阵.证明:
$$\phi(A)=\mathrm{trace}(A),\quad \forall A\in M_n(K).$$

二、设$K[x]$为数域$K$上的一元多项式代数, $I$是$K[x]$的线性子空间,且满足
$$f(x)g(x)\in I,\qquad \forall f(x)\in K[x],g(x)\in I.$$
证明存在$h(x)\in I$,使得$I=\{h(x)f(x)|\forall f(x)\in K[x]\}$.

三、设$K$是一个数域, $V$为$K$上一个$n$维线性空间, $\phi:V\to V$是一个线性变换.证明存在正整数$m$,使得$\mathrm{Im}\phi^m\oplus \mathrm{Ker}\phi^m=V$.

 

四、以$G$记$\mathbb{C}$上行列式为$1$的$2\times 2$矩阵全体.按矩阵的乘法$G$形成一个群.求出$G$的全部正规子群.

 

五、设$A$为实数域上的$m\times n$矩阵,证明必存在实数域上的$n\times m$矩阵$B$,使得$ABA=A,BAB=B$.并请你分析,

(1)这样的$B$唯一吗?

(2)若再加上乘积$AB$和$BA$均要求为对称矩阵,这样的$B$是否仍存在?唯一?给出(1)、(2)的说明.

 


六、设$A$、$B$、$C$为$n$阶矩阵,其中$C=AB-BA$,且它们恰满足$AC=CA$、$BC=CB$.求证: $C$的特征值均为零.

七、设$f_1(x),f_2(x)$是数域$K$上互素的多项式,证明,对次数小于$f_1(x)$的多项式$g_1(x)$和次数小于$f_2(x)$的多项式$g_2(x)$,必存在$g(x)\in K[x]$,使得下述带余除法中的余项分别是$g_1(x)$和$g_2(x)$,
$$
\begin{cases}
g(x)=f_1(x)q_1(x)+g_1(x),\\
g(x)=f_2(x)q_2(x)+g_2(x).
\end{cases}
$$


八、以$M_n(K)$表示数域$K$上的$n$阶方阵全体.设$\phi:M_n(K)\to M_n(K)$是一个线性映射,满足

(1) $\phi(AB)=\phi(A)\phi(B)$\ ($\forall A$、$B\in M_n(K)$和

(2) $\phi(I_n)=I_n$,其中$I_n$是$n$阶单位阵.

证明:存在可逆阵$P\in M_n(K)$,使得$\phi(A)=P^{-1}AP$\ ($\forall A\in M_n(K)$).


九.若$X=\left( \begin{matrix}
A& B\\
C& D\\
\end{matrix} \right)$,其中$A$、$B$、$C$、$D$是$n$阶方阵且两两可交换.证明: $|X|=0$当且仅当$|AD-BC|=0$.


十、二次型$(f,g)=\sum_{i,j=1}^{n}a_{ij}b_{ij}x_iy_j$称为两个二次型$f=\sum_{i,j=1}^{n}a_{ij}x_iy_j,g=\sum_{i,j=1}^{n}b_{ij}x_iy_j$的合成.证明:若$(f,g)$不正定,则$f$、$g$中有一个不正定.

 

 

 


\section{复旦大学2009年数学竞赛分析卷试题}

1.计算$\lim_{n\to+\infty}\int_{0}^{2}\frac{x^n\ln x}{1+x^n} dx$,并说明计算过程合理.

2.设$f(x)$在$[a,b]$上连续,对任何$x\in (a,b)$,极限
$$\lim_{h\to 0} \frac{f(x+h)-f(x-h)}{h}$$
存在且非负.证明: $f(a)\leqslant f(b)$.

3.设$f(x)$在$[a,b]$上Riemann可积,证明$f(x)$在$[a,b]$上某一点连续.


4.设$x_n,y_n\geqslant 0$满足
$$\begin{cases}
x_n^{\frac{4n+5}{n}}+3x_n+y_n=1+\frac{1}{n}, \\
y_n^{\frac{n+2}{n}}+4x_n+3y_n=4-\frac{1}{n}.
\end{cases}$$
证明$\{x_n\},\{y_n\}$收敛并求极限.

5.设$f(x)$在$[0,+\infty)$上连续可导,证明$\lim_{x\rightarrow +\infty}\left( f'\left( x \right) +f\left( x \right) \right) =\ell$
当且仅当$\lim_{x\rightarrow +\infty}f(x)=\ell$且$f'\left( x \right)$在$[0,+\infty)$一致连续.

 

6.设$a_1,a_2>0,a_{n+2}=\frac{1}{a_n}+\frac{1}{a_{n+1}}$.
证明: $\lim_{n\rightarrow +\infty}a_n=\sqrt{2}$.


7.设$n$为自然数.证明:
$$\sum_{\substack{k+j=n\\0\leqslant k,j\leqslant n}}C_{2k}^kC_{2j}^j=4^n.$$

8.设$f(x)$是$[0,1]$上的连续函数, $f(0)=f(1)$, $a$是一个无理数,证明
$$
\lim_{n\rightarrow +\infty}\frac{1}{n}\sum_{k=1}^n{f\left( \left\{ ka \right\} \right)}=\int_0^1{f\left( x \right) dx}.
$$
其中$\{ka\}$表示$ka$的小数部分.

 

\section{复旦大学2009年数学竞赛代数卷试题}

一、设$f(x)$是数域$K$上的一个次数大于$0$的一元多项式.证明$f(x)$是一个不可约多项式$p(x)$的幂(即存在正整数$m$,满足$f(x)=p^m(x)$)的充要条件是:对任意的多项式$g(x)$和$h(x)$,若有$f(x)\mid (g(x)h(x))$,则必有$f(x)\mid g(x)$,或$f(x)\mid h^n(x)$,其中$n$是某个正整数.


二、设$a,b,c$为实数,计算$n$阶行列式
$$
I_n=\left| \begin{matrix}
a& b& 0& \cdots& 0& 0& 0\\
c& a& b& \cdots& 0& 0& 0\\
0& c& a& \cdots& 0& 0& 0\\
\cdots& \cdots& \cdots& \cdots& \cdots& \cdots& \cdots\\
0& 0& 0& \cdots& c& a& b\\
0& 0& 0& \cdots& 0& c& a
\end{matrix} \right|.
$$


三、以$\mathbb{R}_n$记$n$维列向量全体形成的线性空间, $P$为$\mathbb{R}$上$n$阶可逆阵.令
$$
L=\left\{ P^{-1}AP\left| \text{$A$为$\mathbb{R}$上对角线为零的$n$阶上三角阵} \right. \right\}.
$$
设$V$为$\mathbb{R}_n$的一个线性子空间,满足$V=LV$,其中$LV$是由所有
$\{Av|A\in L,v\in V\}$张成的$V$的线性子空间.证明: $V$必为零子空间.

四、设$L$是数域$K$上$n$维线性空间$V$的一些线性变换构成的集合. $V$的一个线性子空间$U$若是$L$中每个线性变换的不变子空间,则称$U$为$L$的不变子空间.当$L$不变子空间只有$V$和$\mathbf{0}$时,称$L$为不可约的.

1.设$L$不可约,且$\psi$是$V$上的与$L$中元素均交换的线性变换,证明$\psi=0$或$\psi$是可逆变换.

2.在$n=2$且$K=\mathbb{C}$时,给出一个不可约的$L$的例子.


五、设$A$为$n$阶实非奇异矩阵.证明存在两个正交阵$P$和$Q$,满足
$$
PAQ=\left( \begin{matrix}
a_1& & & \\
& a_2& & \\
& & \ddots& \\
& & & a_n\\
\end{matrix} \right) ,\quad \text{其中}a_i>0,1\leqslant i\leqslant n.
$$


六、设矩阵$A$相似于Jordan 标准形
$$
J=\left( \begin{matrix}
J_1& 0\\
0& J_2\\
\end{matrix} \right),
$$
其中$J_1,J_2$分别为特征值为$\lambda_1,\lambda_2$的Jordan块.证明:

1.当$\lambda_1\neq\lambda_2$时,与$A$ (乘法)可交换的矩阵必为$A$的多项式;

2.当$\lambda_1=\lambda_2$时,和与$A$ (乘法)可交换的所有矩阵可交换的矩阵必为$A$的多项式.

七、以$G_2$记$\mathbb{C}$上行列式为$1$的$2$阶方阵全体, $N$是$G_2$的一个包含$\{I_2,-I_2\}$的子集(其中$I_2$表示二阶单位阵),满足:

1. $N$关于矩阵的乘法和求逆封闭,即$\forall A,B\in N,AB\in N,A^{-1}\in N$;

2.与$N$中矩阵相似的$2$阶方阵仍在$N$中;

证明: $N=\{I_2,-I_2\}$或$N=G_2$.

八、以$M_n(\mathbb{R})$记$\mathbb{R}$上$n\ (n>1)$阶方阵全体.对$A\in M_n(\mathbb{R})$,及$1\leqslant k\leqslant n$,记$\sigma_k(A)$为$A$的所有$k$阶主子式之和.例如$\sigma_1(A)$为$A$的迹, $\sigma_n(A)$为$A$的行列式等.可以注意到, $\sigma_k(A)$为$A$的项的多项式.现设$\sigma:M_n(\mathbb{R})\to \mathbb{R}$是这样的一个多项式函数,
$$
\sigma \left( \left( a_{ij} \right) \right) =p\left( a_{11},a_{12},\cdots ,a_{1n},\cdots ,a_{n1},\cdots ,a_{nn} \right)
$$
其中$p(\cdots)$是$a_{11},a_{12},\cdots ,a_{1n},\cdots ,a_{n1},\cdots ,a_{nn}$的一个固定多项式.设$\sigma$在相似关系下不变,即
$$
\sigma \left( A \right) =\sigma \left( P^{-1}AP \right),\quad \forall A,P\in M_n(\mathbb{R}),\text{其中$P$可逆},
$$
证明$\sigma$必为$\mathbb{R}$上$\sigma_1,\cdots,\sigma_n$的多项式,即存在一个多项式$F(\sigma_1,\cdots,\sigma_n)$,使得
$$\sigma \left( A \right) =F(\sigma_1\left( A \right),\cdots,\sigma_n\left( A \right)),\quad \forall A\in M_n(\mathbb{R}).$$

%https://www.doc88.com/p-8476065378678.html

 

\section{复旦大学2010年数学竞赛分析卷试题}

一(12\%)、证明:
$$
e^{-2t}=\frac{1}{\sqrt{\pi}}
\int_0^{+\infty}{e^{-y-\frac{t^2}{y}}\frac{dy}{\sqrt{y}}},\quad \forall t>0.
$$

 


二(13\%)、设$f$是$[-1,1]$上的非负连续函数,满足
$$\int_{-1}^{1}xf(x)dx=0,\qquad \int_{-1}^{1}f(x)dx=1.$$
证明
$$\int_{-1}^{1}\int_{-1}^{1}|x+y|f(x)f(y)dxdy\geqslant \int_{-1}^{1}|x|f(x)dx.$$

三(15\%)、定义
$$
F\left( \lambda \right) =\int_{\mathbb{R}}{e^{-\lambda \left( x^2+1-\cos x \right)}\frac{x}{\text{sh}x}dx},\quad \lambda>0.
$$
求$\lim_{\lambda\to +\infty} \sqrt{\lambda}F(\lambda)$.

 

四(15\%)、设$p>1$, $f$在$(0,+\infty)$连续且$\int_{0}^{+\infty} |f(t)|^pdt$收敛.证明:
$$
\left\{ \int_0^{+\infty}{\left[ \frac{1}{x}\int_0^x{\left| f\left( t \right) \right|dt} \right] ^pdx} \right\} ^{\frac{1}{p}}\leqslant \frac{p}{p-1}\left( \int_0^{+\infty}{\left| f\left( t \right) \right|^pdt} \right) ^{\frac{1}{p}}.
$$

 

五(15\%)、设有可数个正数列$\left\{ a_{n}^{\left( k \right)}:n,k\geqslant 1 \right\}$
满足:

1. $\sum_{n\geqslant 1}a_{n}^{\left( k \right)}=1,\quad \forall k\geqslant 1$.


2.记$\mathbb{N}$为正整数集,若对任何$A\subset \mathbb{N}$, $\lim_{k\to+\infty} \sum_{n\in A}a_{n}^{\left( k \right)}$存在.


令$a_n=\lim_{k\to+\infty} a_{n}^{\left( k \right)}$.证明: $\sum_{n\geqslant 1}a_{n}=1$.

 

六(15\%)、设$a_n>0$,且$\sum_{n\geqslant 1}a_{n}=1$.证明
$$
F=\left\{ \sum_{n\in A}{a_n}:A\subset \mathbb{N} \right\}
$$
是一个闭集(注: $A$可以取空集),其中$\mathbb{N}$为正整数集.


七(15\%)、设$f$是$[0,1]$上的右连续函数, $\mathbb{Q}$是$[0,1]$上的有理数集.若$f$沿着$\mathbb{Q}$有左极限,即$\forall x\in (0,1]$,
$$\lim_{y\to x^-,y\in \mathbb{Q}}f(y)$$
存在.证明: $f$在任何点$x\in (0,1]$上有左极限.

 

 


\section{复旦大学2010年数学竞赛代数卷试题}

2010年6月12日

一、求下列$n$阶行列式的值:
$$
\left| \begin{matrix}
a_1& x& x& \cdots& x\\
x& a_2& x& \cdots& x\\
x& x& a_3& \cdots& x\\
\vdots& \vdots& \vdots& & \vdots\\
x& x& x& \cdots& a_n
\end{matrix} \right|.
$$


二、设$A$是$n$阶$(n\geqslant 2)$非零实方阵且满足$A=A^{\ast\ast}$ (其中$A^{\ast}$是$A$的伴随阵, $A^{\ast\ast}$是$A^{\ast}$的伴随阵),求$A$的行列式的值.

三、求解下列线性方程组:
$$
\begin{cases}
ax_1+x_2+x_3=b,\\
x_1+ax_2+x_3=1,\\
x_1+x_2+ax_3=1.
\end{cases}
$$

 

四、设$n$阶循环阵$A=\left( \begin{matrix}
1& 2& \cdots& n-1& n\\
n& 1& \cdots& n-2& n-1\\
\vdots& \vdots& \vdots& \vdots& \vdots\\
3& 4& \cdots& 1& 2\\
2& 3& \cdots& n& 1
\end{matrix} \right) ,A=\left( \begin{array}{c}
A_1\\
A_2\\
\end{array} \right)$
为其分块,其中$A_1$为$k\times n$矩阵, $A_2$为$(n-k)\times n$矩阵, $1\leqslant k\leqslant n-1$.设$V_i$为齐次线性方程组$A_iX=0$的解空间,并将$V_i$看成是有理数域上$n$维列向量空间$\mathbb{Q}^n$的子空间, $i=1,2$.证明: $\mathbb{Q}^n=V_1\oplus V_2$.


五、设$n$阶方阵$A$的全体特征值为$\lambda_1,\lambda_2,\cdots,\lambda_n$,求$A$的伴随阵$A^\ast$的全体特征值.


六、设$A$是$n$阶方阵,证明: $A$的相抵标准型和$A$的Jordan标准型相同的充分必要条件是$A$是幂等阵,即$A^2=A$.


七、设$A$是$m$阶方阵, $B$是$n$阶方阵,并且$A,B$没有公共的特征值.证明:矩阵方程$AX=XB$只有零解$X=0$.


八、设$A$是$m$阶方阵, $B$是$n$阶方阵,并且$A,B$没有公共的特征值.证明:若$A,B$都可以复相似于对角阵,则对任意的$m\times n$矩阵$C$, $(m+n)$阶方阵$\left( \begin{matrix}
A& C\\
0& B\\
\end{matrix} \right)$
也可以复相似于对角阵.


九、设$n\geqslant 2$是自然数, $p\geqslant 3$是素数.证明: $x^n+x+p$是有理数域$\mathbb{Q}$上的不可约多项式.


十、设$A$为$n$阶非异方阵,证明: $A$与所有的$A^k$ ($k$为全体正整数)都相似的充分必要条件是$A$的所有特征值都为$1$.

 

 


\section{复旦大学2010年数学竞赛实变函数卷试题}

一、设$f$是实直线$\mathbb{R}$上的实函数,若有常数$M>0$使得对任何有限个两两不同的实数$x_1,\cdots,x_n$都有$\left|\sum_{i=1}^{n}f(x_i)\right|\leqslant M$,证明集$\{x:f(x)\neq 0\}$是至多可数的.

二、设$E$是实直线$\mathbb{R}$的Lebesgue可测集,且$\mathbf{m}(E)<\infty$.证明:
$$\lim_{n\to\infty}\int_Ee^{inx}dx=0.$$
这里$\mathbf{m}$表示Lebesgue测度.

三、设$f$是$[0,1]$上实的Lebesgue可测函数,并且$\mathbb{Z}$是整数集.证明
$$\lim_{n\to\infty}\int_{0}^{1} |\cos f(x)|^ndx=\mathbf{m}\left(f^{-1}(\pi \mathbb{Z})\right).$$

四、对测度空间$(X,\Sigma,\mu)$,设$f(x)$是$X$上的非负可测函数,且$X$是$\sigma$-有限的.
写$\mu \left( f>t \right) =\mu \left( \left\{ x:f\left( x \right) >t \right\} \right)$,
证明:
$$\int_Xfdu=\int_{0}^{\infty} \mu \left( f>t \right)dt.$$



\section{数列与函数不等式}


\begin{liti}
(2009年高联)求证不等式:
$$
-1<\left( \sum_{k=1}^n{\frac{k}{k^2+1}} \right) -\ln n\leqslant \frac{1}{2},\quad n=1,2,\cdots
$$
\end{liti}
\begin{jieda}
首先证明一个不等式:
\begin{align*}
\frac{x}{1+x}<\ln (1+x)<x,\quad x>0. \tag{1}
\end{align*}
事实上,令
$$h(x)=x-\ln (1+x),\quad g(x)=\ln (1+x)-\frac{x}{1+x}.$$
则对$x>0$,
$$h'(x)=1-\frac{1}{1+x}>0,\quad g'(x)=\frac{1}{1+x}
-\frac{1}{(1+x)^2}=\frac{x}{(1+x)^2}>0.$$
于是
$$h(x)>h(0)=0, \quad g(x)>g(0)=0.$$

在(1)中取$x=\frac{1}{n}$得
\[\frac{1}{n+1}<\ln\left(1+\frac{1}{n}\right)<\frac{1}{n}.\]
令$x_n=\sum_{k=1}^n{\frac{k}{k^2+1}}-\ln n$,则$x_1=\frac{1}{2}$,
\begin{align*}
x_n-x_{n-1} &=\frac{n}{n^2+1}-\ln \left( 1+\frac{1}{n-1} \right)\\
&<\frac{n}{n^2+1}-\frac{1}{n}=-\frac{1}{\left( n^2+1 \right) n}<0.
\end{align*}
因此$x_n<x_{n-1}<\cdots<x_1=\frac{1}{2}$.

又因为
\begin{align*}
\ln n= &\left[ \ln n-\ln \left( n-1 \right) \right] +\left[ \ln \left( n-1 \right) -\ln \left( n-2 \right) \right]\\
&+\cdots +\left[ \ln 2-\ln 1 \right] =\sum_{k=1}^{n-1}{\ln \left( 1+\frac{1}{k} \right)}.
\end{align*}
从而
\begin{align*}
x_n &=\sum_{k=1}^n{\frac{k}{k^2+1}}-\sum_{k=1}^{n-1}{\ln \left( 1+\frac{1}{k} \right)}\\
&=\sum_{k=1}^{n-1}{\left[ \frac{k}{k^2+1}-\ln \left( 1+\frac{1}{k} \right) \right]}+\frac{n}{n^2+1}\\
&>\sum_{k=1}^{n-1}{\left[ \frac{k}{k^2+1}-\frac{1}{k} \right]}=-\sum_{k=1}^{n-1}{\frac{1}{\left( k^2+1 \right) k}}\\
&\geqslant -\sum_{k=1}^{n-1}{\frac{1}{\left( k+1 \right) k}}=-1+\frac{1}{n}>-1.
\end{align*}
\end{jieda}

 

\begin{liti}
(2010年湖北高考)


\begin{example}[2010年湖北,难度: \score{3}{5}]
已知函数$f\left( x \right) = ax + \dfrac{b}{x} + c$ ($a > 0$)的图象在$\left( {1 , f\left( 1 \right)} \right)$处的切线方程为$y = x-1$.

(1)用$a$表示出$b$, $c$;

(2)若$f\left( x \right) \geqslant \ln x$在$\left[ {1, + \infty } \right)$上恒成立,求$a$的取值范围;

(3)证明: $1 + \dfrac{1}{2} + \dfrac{1}{3} + \cdots + \dfrac{1}{n} > \ln \left( {n + 1} \right) + \dfrac{n}{{2\left( {n + 1} \right)}}$ ($n \in {\mathbb{N}^\ast }$).
\end{example}


\end{liti}
\begin{jieda}
(I) $f'(x)=a-\frac{b}{x^2}$,则有$\begin{cases}
f(1)=a+b+c=0, \\
f'(1)=a-b=1,
\end{cases}$
解得$\begin{cases}
b=a-1, \\
c=1-2a.
\end{cases}$

(II)由(I)知, $f(x)=ax+\frac{a-1}{x}+1-2a$.

令$g(x)=f(x)-\ln x=ax+\frac{a-1}{x}+1-2a-\ln x,x\in [1,+\infty)$,则$g(1)=0$,
$$
g'\left( x \right) =a-\frac{a-1}{x^2}-\frac{1}{x}=\frac{ax^2-x-\left( a-1 \right)}{x^2}=\frac{a\left( x-1 \right) \left( x-\frac{1-a}{a} \right)}{x^2},
$$

(i)当$0<a<\frac{1}{2}$时, $\frac{1-a}{a}>1$.

若$1<x<\frac{1-a}{a}$,则$g'(x)<0$, $g(x)$是减函数,所以$g(x)<g(1)=0$,即$f(x)<\ln x$.

故$f(x)\geqslant\ln x$在$[1,+\infty)$上不恒成立.

(ii)当$a\geqslant\frac{1}{2}$时, $\frac{1-a}{a}\leqslant 1$.

若$x>1$,则$g'(x)>0$, $g(x)$是增函数,所以$g(x)>g(1)=0$,即$f(x)>\ln x$.

故当$x\geqslant 1$时, $f(x)\geqslant \ln x$.

综上所述,所求$a$的取值范围为$\left[ \frac{1}{2},+\infty \right)$.

(III) \textbf{解法一:}由(II)知:

当$a\geqslant\frac{1}{2}$时,有$f(x)\geqslant \ln x\ (x\geqslant 1)$.

令$a=\frac{1}{2}$,有$f\left( x \right) =\frac{1}{2}\left( x-\frac{1}{x} \right) \geqslant \ln x\ (x\geqslant 1)$.

且当$x>1$时, $\frac{1}{2}\left( x-\frac{1}{x} \right)> \ln x$.

令$x=\frac{k+1}{k}$,有
$$
\ln \frac{k+1}{k}<\frac{1}{2}\left[ \frac{k+1}{k}-\frac{k}{k+1} \right] =\frac{1}{2}\left[ \left( 1+\frac{1}{k} \right) -\left( 1-\frac{1}{k+1} \right) \right],
$$

$$
\ln \left( k+1 \right) -\ln k<\frac{1}{2}\left( \frac{1}{k}+\frac{1}{k+1} \right),\quad k=1,2,3,\cdots,n.
$$
将上述$n$个不等式依次相加得
$$
\ln \left( n+1 \right) <\frac{1}{2}+\left( \frac{1}{2}+\frac{1}{3}+\cdots +\frac{1}{n} \right) +\frac{1}{2\left( n+1 \right)},
$$
整理得
$$1+\frac{1}{2}+\frac{1}{3}+\cdots +\frac{1}{n}>\ln \left( n+1 \right)+\frac{n}{2\left( n+1 \right)}.$$

\textbf{解法二:}用数学归纳法证明.

(1)当$n=1$时,左边$=1$,右边$=\ln 2+\frac{1}{4}<1$,不等式成立.

(2)假设$n=k$时,不等式成立,就是
$$
1+\frac{1}{2}+\frac{1}{3}+\cdots +\frac{1}{k}>\ln \left( k+1 \right) +\frac{k}{2\left( k+1 \right)}.
$$
那么
\begin{align*}
&1+\frac{1}{2}+\frac{1}{3}+\cdots +\frac{1}{k}+\frac{1}{k+1}\\
&>\ln \left( k+1 \right) +\frac{k}{2\left( k+1 \right)}+\frac{1}{k+1}=\ln \left( k+1 \right) +\frac{k+2}{2\left( k+1 \right)}.
\end{align*}
由(II)知:

当$a\geqslant\frac{1}{2}$时,有$f(x)\geqslant \ln x\ (x\geqslant 1)$.

令$a=\frac{1}{2}$,有$f\left( x \right) =\frac{1}{2}\left( x-\frac{1}{x} \right) \geqslant \ln x\ (x\geqslant 1)$.

令$x=\frac{k+2}{k+1}$,得:
$$
\frac{1}{2}\left( \frac{k+2}{k+1}-\frac{k+1}{k+2} \right) \geqslant \ln \frac{k+2}{k+1}=\ln \left( k+2 \right) -\ln \left( k+1 \right).
$$
所以
$$
\ln \left( k+1 \right) +\frac{k+2}{2\left( k+1 \right)}\geqslant \ln \left( k+2 \right) +\frac{k+1}{2\left( k+2 \right)}.
$$
所以
$$
1+\frac{1}{2}+\frac{1}{3}+\cdots +\frac{1}{k}+\frac{1}{k+1}\geqslant \ln \left( k+2 \right) +\frac{k+1}{2\left( k+2 \right)}.
$$
这就是说,当$n=k+1$时,不等式也成立.

根据(1)和(2),可知不等式对任何$n\in \mathbb{N}^\ast$都成立.
\end{jieda}

 



\section{有理逼近}

丢番图逼近

Ivan Niven,无理数,丢番图逼近


每个无理数$\theta$都可用有理数逼近到我们所需的任意精确度.事实上,如果我们去掉$\theta$的小数点后第$n$位之后的数字,那么我们就得到了一个和$\theta$的误差小于$10^{-n}$的有理数.然而,通过这种截取小数所得到的有理数的分母却可能很大,例如,如果取
$$\theta=\pi-3=0.141\ 592\ 653\cdots$$
那么其前$5$个逼近小数将分别是$0.1,0.14,0.141,0.1415,0.14159$.如果把它们写成$\frac{a}{b}$的形式,其中$a$和$b$是互素的整数,那么这些近似的有理数将是
$$\frac{1}{10},\quad \frac{7}{50},\quad \frac{141}{1\ 000},\quad \frac{283}{2\ 000},\quad \frac{14\ 159}{100\ 000}$$
另一方面,分数$\frac{1}{7}=0.142\ 857\cdots$和$\theta$的误差要小于$\frac{2}{1\ 000}$,并且非常接近于近似于$\theta$的$\frac{141}{1\ 000}$,但是它的分母是$7$,这是一个和$1\ 000$比起来很小的数.

这个例子建议我们提出以下问题:给定一个实数$\theta$,是否存在一个有理数$\frac{h}{k}$,使得它是$\theta$的一个很好的逼近,但是其分母$k$却不太大?

显然,这是一个模糊的问题,由于术语“好的逼近”和“不太大”的意义都是模糊的.在我们把问题提的更确切之前,我们首先用稍微不同的方式提出问题.


如果$\theta-\frac{h}{k}$是一个很小的数,那么$\frac{k\theta-h}{k}$将也是一个很小的数,为了使得这个去掉$k$后会变大的数也是很小的,那么分子$k\theta-h$也必须是一个很小的数.因此,我们可以提出以下问题:

对任意给定的实数$\theta$和$\varepsilon>0$,是否存在整数$h$和$k$使得$|k\theta-h|<\varepsilon$?

 

下面的Dirichlet (狄利克雷)定理肯定地回答了这个问题.


\begin{thm}
对任意给定的实数$\theta$和预先给定的任意整数$N$,必存在整数$h$和$k$,使得$0<k\leqslant N$并且
$$|k\theta-h|<\frac{1}{N}.$$
\end{thm}
\begin{jieda}
设$\{x\}=x-[x]$表示$x$的小数部分,考虑$N+1$个实数
$$0,\{\theta\},\{2\theta\},\cdots,\{N\theta\}$$
所有这些数都位于半开半闭区间$0\leqslant \{m\theta\}<1$之中.现在把单位区间分成$N$个相等的,长度都是$\frac{1}{N}$的半开子区间.那么其中某个子区间必至少包含两个小数部分,比如说是$\{a\theta\}$和$\{b\theta\}$,其中$0\leqslant a<b\leqslant N$,因此有
$$|\{b\theta\}-\{a\theta\}|<\frac{1}{N}.$$
但是
$$\{b\theta\}-\{a\theta\}=b\theta-[b\theta]-a\theta+[a\theta]
=(b-a)\theta-([b\theta]-[a\theta]).$$
因此,如果设
$$k=b-a,\qquad h=[b\theta]-[a\theta],$$
那么就有
$$|k\theta-h|<\frac{1}{N},\quad 0<k\leqslant N.$$
这就证明了定理.
\end{jieda}


\textbf{注记.}任给$\varepsilon>0$,我们可选取$N>\frac{1}{\varepsilon}$,
则有$|k\theta-h|<\varepsilon$.


下一个定理说明我们可以把$h$和$k$选成是互素的.

\begin{thm}
对任意给定的实数$\theta$和预先给定的任意整数$N$,必存在互素的整数$h$和$k$,使得$0<k\leqslant N$和
$$|k\theta-h|<\frac{1}{N}.$$
\end{thm}
\begin{jieda}
根据上述定理,存在一对整数$h',k'$使得$0<k'\leqslant N$,并且
$$\left|\theta-\frac{h'}{k'}\right|<\frac{1}{Nk'}.$$
设$d=(h',k')$.如果$d=1$,那么定理已成立.

如果$d>1$,令$h=\frac{h'}{d},k=\frac{k'}{d}$,那么$(h,k)=1$,
并且$k<k'\leqslant N$.因此有$\frac{1}{k'}<\frac{1}{k}$,因而由上式就得出
$$\left|\theta-\frac{h}{k}\right|=\left|\theta-\frac{h'}{k'}\right|
<\frac{1}{Nk'}<\frac{1}{Nk}.$$
由此就得出
$$|k\theta-h|<\frac{1}{N}.$$
\end{jieda}

 

现在,我们用一种稍微有些弱的形式重新叙述上面的结果,在这种叙述方式中将不出现整数$N$.

 

\begin{thm}
对任意给定的实数$\theta$,存在整数$h$和$k$,使得$k>0,(h,k)=1$并且
$$\left|\theta-\frac{h}{k}\right|<\frac{1}{k^2}.$$
\end{thm}
\begin{jieda}
根据上面定理,存在整数$h$和$k$使得$0<k\leqslant N$, $(h,k)=1$并且
$$|k\theta-h|<\frac{1}{N},$$
所以
$$\left|\theta-\frac{h}{k}\right|<\frac{1}{kN}\leqslant\frac{1}{k^2}.$$
\end{jieda}

 


\begin{thm}
(Hurwitz不等式,难度: \score{3}{5})对任意实数$\theta$,必存在无穷多个有理数$\frac{h}{k}$,使得
$$
\left| \theta -\frac{h}{k} \right|<\frac{1}{\sqrt{5}k^2},
$$
其中$\sqrt{5}$是最佳常数.
\end{thm}
\begin{jieda}

\end{jieda}

接下来证明一个称之为Liouville (刘维尔)定理的结果,这个定理说明Hurwitz不等式分母中的$k^2$不可能换成$k^3$或更高的幂.


\begin{thm}(刘维尔定理)
设$\theta$是一个任意的代数数,其次数$n\geqslant 2$.则存在仅依赖于$\theta$的正常数$C(\theta)$,使得对所有的整数$h$和$k>0$都成立
$$
\left| \theta -\frac{h}{k} \right|>\frac{C(\theta)}{k^n}.
$$
\end{thm}
\begin{jieda}
由于$\theta$是一个$n$次代数数,因此$\theta$必是某个整系数$n$次多项式$f(x)$
的根,不妨设
$$f(x)=a_nx^n+a_{n-1}x^{n-1}+\cdots+a_1x+a_0\quad (a_n\neq 0)$$
其中$f(x)$在有理数域上是不可约的,因此$f(x)$没有有理根,因而对每个有理数$\frac{h}{k}$都有$f\left( \frac{h}{k} \right) \neq 0$.

现在,应用微积分学中的拉格朗日中值定理就得出
$$
f\left( \frac{h}{k} \right) =f\left( \frac{h}{k} \right) -f\left( \theta \right) =f'\left( \xi \right) \left( \frac{h}{k}-\theta \right),
$$
其中$\xi$位于$\theta$和$\frac{h}{k}$之间.我们有
$$
f\left( \frac{h}{k} \right) =\sum_{m=0}^n{a_m\left( \frac{h}{k} \right) ^m}=\frac{N}{k^n},
$$
其中$N$是一个非零整数,因此就有
$$
\left| f\left( \frac{h}{k} \right) \right|\geqslant \frac{1}{k^n}.
$$
又设$d=\left| \theta -\frac{h}{k} \right|$.

如果$d> 1$,那么显然就有$\left| \theta -\frac{h}{k} \right|>1>\frac{1}{k^n}$,
于是取$C(\theta)=1$即可.

因此不妨设$d<1$ (由于$\theta$是无理数,所以$d$不可能等于$1$),因为$\xi$位于$\theta$和$\frac{h}{k}$之间并且$d<1$,所以有$|\xi-\theta|<1$,因此
$$|\xi|=|\theta+\xi-\theta|\leqslant|\theta|+|\xi-\theta|<|\theta|+1$$
因而
$$
\left| f'\left( \xi \right) \right|\leqslant A\left( \theta \right) <1+A\left( \theta \right),
$$
其中$A\left( \theta \right)$表示$\left| f'\left( x \right) \right|$在区间$|x|\leqslant |\theta|+1$上的最大值.由此就得出
$$
\left| \theta -\frac{h}{k} \right|=\frac{\left| f\left( \frac{h}{k} \right) \right|}{\left| f\left( \xi \right) \right|}\geqslant \frac{\frac{1}{k^n}}{1+A\left( \theta \right)}\geqslant \frac{C\left( \theta \right)}{k^n},
$$
其中$C\left( \theta \right)=\frac{1}{1+A\left( \theta \right)}$.


\textbf{证法二.}设$P(x)$是$\theta$的最小多项式.通过对$P(x)$进行因式分解,我们可以得到一个在$\mathbb{Z}[x]$中不可约且首项系数为正的整系数$n$次多项式.


$$f(x)=a_nx^n+a_{n_1}x^{n-1}+\cdots+a_1x+a_0\in \mathbb{Z}[x]$$
为此多项式.我们有时称之为$\theta$在$\mathbb{Z}$上的最小多项式.则
$$
\left| f\left( \theta \right) -f\left( \frac{h}{k} \right) \right|=\left| f\left( \frac{h}{k} \right) \right|=\left| \frac{a_nh^n+a_{n-1}h^{n-1}k+\cdots +a_0k^n}{k^n} \right|\geqslant \frac{1}{k^n}.
$$
若$\theta=\theta_1,\cdots,\theta_n$为$f$的根,令$M$为$|\theta_i|$中的最大值.若$\left| \frac{h}{k} \right|\geqslant 2M$,则
$$
\left| \frac{h}{k}-\theta \right|\geqslant M\geqslant \frac{M}{k^n}.
$$
若$\left| \frac{h}{k} \right|\leqslant 2M$,则
$$
\left| \theta_i-\frac{h}{k} \right|\leqslant 3M.
$$
因此
$$
\left| \theta -\frac{h}{k} \right|\geqslant \frac{1}{\left| a_n \right|k^n\prod_{m=2}^n{\left| \theta _m-\frac{h}{k} \right|}}\geqslant \frac{1}{\left| a_n \right|\left( 3M \right) ^{n-1}k^n}.
$$
于是取
$$
C\left( \theta \right) =\min \left\{ M,\frac{1}{\left| a_n \right|\left( 3M \right) ^{n-1}} \right\}
$$
即可.
\end{jieda}

称不是代数数的实数为超越数,通过简单的计数即可知道必存在超越数.

事实上,所有代数数的集合是可数的,而所有实数的集合是不可数的,因此超越数不仅存在,而且构成一个不可数集合.

一般来说,要证明某个特殊的具体的数,例如$e$或者$\pi$为超越数是困难的,但是刘维尔定理可以用来证明那种可用有理数充分逼近的无理数必定是超越的,称那种数是刘维尔数,其定义如下:

定义1.称一个实数是一个刘维尔数,如果对每个整数$r\geqslant 1$,都存在整数$h_r$和$k_r$,使得$k_r>0$,并且
$$
0<\left| \theta -\frac{h_r}{k_r} \right|>\frac{1}{k_r^r}.
$$

\begin{thm}
刘维尔数都是超越数.
\end{thm}
\begin{jieda}
假如刘维尔数$\theta$是一个$n$次代数数,那么它将满足不等

$$
0<\left| \theta -\frac{h_r}{k_r} \right|>\frac{1}{k_r^r},
$$
并且对每一个$r\geqslant 1$成立
$$
\left| \theta -\frac{h_r}{k_r} \right|>\frac{C(\theta)}{k_r^n}.
$$
其中$C(\theta)$是常数.因此
$$0<\frac{C(\theta)}{k_r^n}<\frac{1}{k_r^r}$$

$$0<C(\theta)<\frac{1}{k_r^{r-n}}$$
当$r$充分大时,上式与$C(\theta)$是一个正的常数相矛盾.
\end{jieda}

 

\begin{liti}
数$\theta=\sum_{m=1}^{\infty}\frac{1}{10^{m!}}$就是一个刘维尔数,因此是超越数.
\end{liti}
\begin{jieda}
事实上,对任意$r\geqslant 1$,我们可取$k_r=10^{r!}$以及
$$h_r=k_r\sum_{m=1}^r\frac{1}{10^{m!}}$$
这时,我们就有
\begin{align*}
0& <\theta -\frac{h_r}{k_r}=\sum_{m=r+1}^{\infty}{\frac{1}{10^{m!}}}\le \frac{1}{10^{\left( r+1 \right) !}}\sum_{m=0}^{\infty}{\frac{1}{10^m}}\\
&=\frac{\frac{10}{9}}{10^{\left( r+1 \right) !}}=\frac{1}{k_{r}^{r}}\cdot \frac{\frac{10}{9}}{10^r}<\frac{1}{k_{r}^{r}}.
\end{align*}
得证.
\end{jieda}


\textbf{注记.}同理可证$\sum_{m=1}^{\infty}\frac{a_m}{10^{m!}}$是超越数,其中对无限多个$m$有$a_m=0$或$a_m=1$.

 

 

 

参考:数论中的模函数与狄利克雷级数

 

\begin{liti}
(难度: \score{3}{5})
设$P(x)$是实系数多项式,且对所有$x\in \mathbb{R}$有$P(x)\geqslant 0$.求证:存在实系数多项式$Q_1(x)$和$Q_2(x)$,使得对所有$x$有
$$P(x)=Q_1^2(x)+Q_2^2(x).$$
\end{liti}
\begin{jieda}
证明由题设$P(x)$可以表示为如下形式:
$$
c\prod_{k=1}^n{\left( x^2+p_kx+q_k \right)},
$$
其中$c,p_k,q_k$是实数,且$c\geqslant 0,p_k^2-4q_k\leqslant 0,k=1,2,\cdots,n$.

注意到$x^2+p_kx+q_k$可以表示为两个式子的平方和
$$
\left( x+\frac{p_k}{2} \right) ^2+\left( \frac{\sqrt{4q_k-p_{k}^{2}}}{2} \right) ^2=u_{k}^{2}\left( x \right) +v_{k}^{2}\left( x \right),
$$
所以只需证明
$$
\left[ u_{1}^{2}\left( x \right) +v_{1}^{2}\left( x \right) \right] \left[ u_{2}^{2}\left( x \right) +v_{2}^{2}\left( x \right) \right] \cdots \left[ u_{n}^{2}\left( x \right) +v_{n}^{2}\left( x \right) \right]
$$
可以表示为两个实系数多项式的平方和.这可由数学归纳法和斐波那契恒等式导出.
\end{jieda}


\begin{liti}
(难度: \score{3}{5})
证明:对任意$n\in N^\ast$,存在一个首项系数为$1$的$n$次整系数多项式$P(x)$,使得$2\cos n\varphi=P(2\cos\varphi)$,这里$\varphi$为任意实数.
\end{liti}
\begin{jieda}
当$n=1$时,取$P(x)=x$即可;当$n=2$时, $2\cos2\varphi=(2\cos\varphi)^2-2$,命题也成立.

假设命题对$n=k$和$k+1$成立,即存在首项系数为$1$的整系数多项式$f(x)$和$g(x)$,使得
$$2\cos k\varphi=f(2\cos \varphi),\quad 2\cos(k+1)\varphi=g(2\cos\varphi).$$
其中$f$、$g$的次数分别为$k$和$k+1$.

下面考虑$n=k+2$的情形.注意到
\begin{align*}
2\cos(k+2)\varphi &=2\cos[(k+1)\varphi+\varphi]
=2\cos(k+1)\varphi\cos\varphi
-2\sin(k+1)\varphi\sin\varphi.\tag*{\ding{172}}\\
2\cos k\varphi &=2\cos[(k+1)\varphi-\varphi]
=2\cos(k+1)\varphi\cos\varphi+2\sin(k+1)\varphi\cos\varphi. \tag*{\ding{173}}
\end{align*}
将\ding{172}与\ding{173}相加,得
$$2\cos(k+2)\varphi+2\cos k\varphi=4\cos(k+1)\varphi\cos\varphi.$$
利用归纳假设,可知
$$2\cos(k+2)\varphi=(2\cos \varphi)g(2\cos \varphi)-f(2\cos \varphi).$$
故令$h(x)=xg(x)-f(x)$ (易知$h(x)$是一个首项系数为$1$的整系数多项式),就有
$$2\cos(k+2)\varphi=h(2\cos \varphi).$$
命题对$k+2$成立.

所以,命题成立.
\end{jieda}

 

\begin{liti}
(难度: \score{3}{5})
平面直角坐标系中格点正多边形只有一种,也就是正方形.
\end{liti}
这个命题可以用无穷递降证明,今天用如下定理给出证明.
\begin{jieda}
Niven定理:若$0\leqslant \theta\leqslant \frac{\pi}{2}$且$\frac{\theta}{\pi}$以及$\sin \theta$同为有理数,则有$\theta \in \left\{ 0,\frac{\pi}{6},\frac{\pi}{2} \right\}$.

原命题等价于证明:若$0\leqslant \theta\leqslant \frac{\pi}{2}$且$\frac{\theta}{\pi}$以及$\cos \theta$同为有理数,则有$\theta \in \left\{ 0,\frac{\pi}{3},\frac{\pi}{2} \right\}$.

利用如下引理:对任意正整数$n\geqslant 1$,存在首一$n$次整系数多项式$P_n(x)$,使得
$$P_n(2 \cos t)=2\cos (nt).$$

由于$\frac{\theta}{\pi}\in \mathbb{Q}$,设$\frac{\theta}{\pi}=\frac{2k}{n}$,其中$k,n\in \mathbb{Z},n\geqslant 1$.又$\cos\theta\in \mathbb{Q}$,令$c=2\cos\theta$,根据引理
$$
P_n\left( c \right) =P_n\left( 2\cos \frac{2k\pi}{n} \right) =2\cos \left( n\cdot \frac{2k\pi}{n} \right) =2\cos \left( 2k\pi \right) =2.
$$
注意到$c\in \mathbb{Q}$,这表明$c$是$P_n\left( x \right)-2=0$的有理根,又$P_n\left( x \right)$是首一的整系数多项式,从而$c$必然是整数.再由$|c|=|2\cos t|\leqslant 2$,因此$c\in\{-2,-1,0,1,2\}$.
又$0\leqslant \theta\leqslant \frac{\pi}{2}$,故$c\in\{0,1,2\}$,这正好对应着$\theta \in \left\{ 0,\frac{\pi}{3},\frac{\pi}{2} \right\}$.证毕.

若存在格点正$n$边形($n>4$),则易知其中心是有理点,其中心角记为$\theta=\frac{2\pi}{n}$.将该正多边形的中心平移至原点,则其所有顶点均为有理点.设其中两个相邻的顶点为$A_1(a,b),A_2(c,d)$,则$OA_1=OA_2=\sqrt{a^2+b^2}
=\sqrt{c^2+d^2}=r$,从而$r^2\in \mathbb{Q}$,则由余弦定理可知
$$\cos\theta=\frac{2r^2-|A_1A_2|^2}{2r^2}\in \mathbb{Q}.$$
再由正弦面积公式可知
$$\sin\theta=\frac{|ad-bc|}{r^2}\in \mathbb{Q}.$$
注意到$\frac{\theta}{\pi}\in \mathbb{Q}$,并且$0<\theta< \frac{\pi}{2}$,由Niven定理可知这不可能,矛盾.因此不存在格点正$n$边形($n>4$).
\end{jieda}


\begin{liti}
(难度: \score{3}{5})
设$a,b\in (0,1)$且$a\geqslant b$.证明: $a^a+b^b\geqslant a^b+b^a$.
\end{liti}
\begin{jieda}
首先证明一个引理.

\textbf{引理.}当$x\in(-1,+\infty),t>1$时, $(1+x)^t\geqslant 1+tx$.

\textbf{证明.}构造函数
$$f(x)=(1+x)^t-1-tx,$$

$$f'(x)=t(1+x)^{t-1}-t=t[(1+x)^{t-1}-1].$$
令$f'(x)\geqslant 0$,则$x\geqslant 0$,故$f(x)$在$(-1,0]$上单调递减,在$[0,+\infty)$上单调递增,于是$f(x)\geqslant f(0)=0$,
即$(1+x)^t\geqslant 1+tx$.

再来看原题的证明:

\textbf{证明.}原不等式等价于当$a,b\in (0,1)$且$a\geqslant b$时, $a^a-a^b\geqslant b^a-b^b$.

构造函数$g(x)=x^a-x^b\ (x\in (0,1)$,则
$$
g\left( x \right) =ax^{a-1}-bx^{b-1}=bx^{b-1}\left( \frac{a}{b}x^{a-b}-1 \right).
$$
下面证明$g(x)$在$[b,a]$上单调递增.事实上, $g'(x)\geqslant 0\Leftrightarrow x^{a-b}\geqslant \frac{b}{a}$,此时只需证明$b^{a-b}\geqslant \frac{b}{a}$,即只需证$a\geqslant b^{1-a+b}\Leftrightarrow a^{\frac{1}{1-a+b}}\geqslant b$.
当$a=b$时显然成立,当$a>b$时, $\frac{1}{1-a+b}>1,a-b>0$.由引理可得
$$
a^{\frac{1}{1-a+b}}=\left( 1+a-1 \right) ^{\frac{1}{1-a+b}}\ge 1+\frac{a-1}{1-a+b}=\frac{b}{1-a+b}>b,
$$
则$g(x)$在$[b,a]$上为增函数,原不等式得证.
\end{jieda}

 

\begin{liti}
(难度: \score{3}{5})
设$\{a_n\}$为非增的正数列,求证:若对$n\geqslant 2001,na_n<1$,则对任意自然数$m\geqslant 2001,x\in \mathbb{R}$,有$\left| \sum_{k=2001}^m{a_k\sin kx} \right|\leqslant 1+\pi$.
\end{liti}
\begin{jieda}
记$f(x)=\left| \sum_{k=2001}^m{a_k\sin kx} \right|$,则$f(x)$是以$2\pi$为周期的周期函数,且又为偶函数, $f(0)=f(\pi)=0$,问题对$x\in
\mathbb{R}$归结为只要$x\in (0,\pi)$内证明$f(x)=\left| \sum_{k=2001}^m{a_k\sin kx} \right|\leqslant 1+\pi$即可.

对于任意取定$x\in (0,\pi)$,取$n\in \mathbb{N}$使$n\leqslant \frac{\sqrt{\pi}}{x}<n+1$,则
$$
\left| \sum_{k=2001}^m{a_k\sin kx} \right|\leqslant \left| \sum_{k=2001}^n{a_k\sin kx} \right|+\left| \sum_{k=n+1}^m{a_k\sin kx} \right|.
$$
(规定$n\leqslant 2001$时$\sum_{k=2001}^n{a_k\sin kx}=0$, $n\geqslant m$时$\sum_{k=n+1}^m{a_k\sin kx}=0$.)

由$|\sin x\leqslant x$得
$$
\left| \sum_{k=2001}^n{a_k\sin kx} \right|\le \sum_{k=2001}^n{\left| a_k\sin kx \right|}\le \sum_{k=2001}^n{\frac{1}{k}\cdot kx}=\left( n-2000 \right) x<nx\le \sqrt{\pi}.
$$

记$s_k=\sum_{k=n+1}^R\sin kx\ (k=n+1,n+2,\cdots,m)$,可证$|s_k|\leqslant\frac{1}{\sin\frac{x}{2}}$.

由Abel不等式有
\begin{align*}
\left| \sum_{k=n+1}^m{a_k\sin kx} \right|&\leqslant \frac{1}{\sin \frac{x}{2}}a_{n+1}\le \frac{1}{n+1}\frac{1}{\sin \frac{x}{2}}\\
&<\frac{1}{n+1}\cdot \frac{1}{\frac{2}{\pi}\cdot \frac{x}{2}}<\frac{\pi}{x\left( n+1 \right)}<\sqrt{\pi},
\end{align*}
所以
$$
f\left( x \right) <\sqrt{\pi}+\sqrt{\pi}=2\sqrt{\pi}<1+\pi.
$$
\end{jieda}

 

posted on 2022-02-02 16:39  Eufisky  阅读(888)  评论(0编辑  收藏  举报

导航